You are on page 1of 83

Shoulder and Elbow Self-Assessment Examination AAOS 2014

Question 1 of 105
A 45-year-old man who had gout in his foot 2 years ago has a 3-day history of elbow
pain without an injury. The pain is diffuse, constant, and worse with any movement.
Examination shows motion from 20 degrees to 90 degrees. There is no erythema
around his elbow, he has no fever, and a sensory and motor examination of his arm is
unremarkable. Radiographs only show an effusion. The patient’s uric acid level is
within defined limits. What is the next diagnostic step?
1- Elbow joint aspiration
2- MRI scan
3- Splint for 2 weeks and repeat examination
4- Sedimentation rate and C-reactive protein level

PREFERRED RESPONSE: 1- Elbow joint aspiration


DISCUSSION
The best way to make the diagnosis is to aspirate the fluid from the joint and send it to
the laboratory for a cell count and crystal search. This will allow for the diagnosis of
an infection, gout, or pseudogout. An MRI scan will confirm the examination finding
of an effusion, but it will not reveal the cause of an inflammatory effusion. If the
patient has chronic gout, an MRI scan or ultrasound can aid in diagnosis if gout tophi
are seen. A splint will help relieve the pain from the effusion and the effusion may
resolve on its own, but it will not contribute to a diagnosis. Sedimentation rate and C-
reactive protein level will help to rule out an infection, but they are not as diagnostic
as an aspiration.
RECOMMENDED READINGS
1- Orzechowski NM, Mason TG. Seronegative inflammatory arthritis. In: Morrey BF, Sanchez-Sotelo J, eds. The
Elbow and Its Disorders. 4th ed. Philadelphia, PA: Saunders-Elsevier; 2009:1039-1041.
2- de Ávila Fernandes E, Kubota ES, Sandim GB, Mitraud SA, Ferrari AJ, Fernandes AR. Ultrasound features of
tophi in chronic tophaceous gout. Skeletal Radiol. 2011 Mar;40(3):309-15. Epub 2010 Jul 31. PubMed:
20676636. View Abstract at PubMed

Question 2 of 105
A 65-year-old man who underwent an uncomplicated reverse total shoulder
arthroplasty (rTSA) to treat rotator cuff arthropathy 2 years ago has a routine follow-
up visit in your clinic. A radiograph at 2-year followup is shown in Figure 2. He
denies shoulder pain and dysfunction and constitutional symptoms, and his clinical
examination findings are benign. Based upon the present radiologic evaluation, what is
the next most appropriate step?
Shoulder and Elbow Self-Assessment Examination AAOS 2014

1. Revision rTSA
2. Conversion to hemiarthroplasty
3. Continued observation
4. Infection work-up with screening labs and joint
aspiration

PREFERRED RESPONSE: 3- Continued observation


DISCUSSION
Based upon the patient’s clinical examination and symptoms, continued observation is
appropriate. The remaining options are not indicated. The radiograph reveals scapular
notching, one of the more common complications specific to rTSA. Notching is
caused by repeated contact between the humeral component and/or humerus and the
inferior pillar of the scapular neck. Generation of particulate debris from this
interaction can result in osteolysis with the potential for screw and base plate failure.
The overall incidence of notching has been reported to be between 51% and 96%. This
nearly ubiquitous finding has been attributed to implant positioning, altered glenoid
and humeral anatomy, and duration of implantation. Recent studies that indicate
increased lateral offset, increased glenosphere size, and inferior positioning of the base
plate may reduce the incidence of scapular notching.
RECOMMENDED READINGS
1. Cheung E, Willis M, Walker M, Clark R, Frankle MA. Complications in reverse total shoulder arthroplasty. J
Am Acad Orthop Surg. 2011 Jul;19(7):439-49. Review. PubMed PMID: 21724923.View Abstract at PubMed
2. Lévigne C, Boileau P, Favard L, Garaud P, Molé D, Sirveaux F, Walch G. Scapular notching in reverse shoulder
arthroplasty. J Shoulder Elbow Surg. 2008 Nov-Dec;17(6):925-35. Epub 2008 Jun 16. PubMed PMID:
18558499. View Abstract at PubMed
3. Gutiérrez S, Levy JC, Frankle MA, Cuff D, Keller TS, Pupello DR, Lee WE 3rd. Evaluation of abduction range
of motion and avoidance of inferior scapular impingement in a reverse shoulder model. J Shoulder Elbow Surg.
2008 Jul-Aug;17(4):608-15. Epub 2008 Mar 6. PubMed: 18325795.View Abstract at PubMed

Question 3 of 105
A 61-year-old right-hand-dominant woman sustains a fall down 3 stairs, resulting in a
left anteroinferior dislocation and noncomminuted greater tuberosity fracture. A
closed glenohumeral reduction with intravenous sedation is performed in the
emergency department. After reduction, the greater tuberosity fragment remains
displaced by 2 mm. What is the most appropriate treatment?
1- Open reduction internal fixation with transosseous sutures
2- Arthroscopic fixation using a suture bridge technique
3- Nonsurgical treatment with early passive range of motion
Shoulder and Elbow Self-Assessment Examination AAOS 2014

4- Nonsurgical treatment with sling immobilization for 4 weeks

PREFERRED RESPONSE: 3- Nonsurgical treatment with early passive range of


motion

DISCUSSION
Greater tuberosity fractures and rotator cuff tears associated with a traumatic
dislocation are more commonly seen in women older than age 60. Greater tuberosity
fractures that are displaced less than 5 mm in the general population and less than 3
mm in laborers and professional athletes can be treated successfully without surgery.
Early passive range of motion is important to avoid the complication of stiffness.
RECOMMENDED READINGS
1- George MS. Fractures of the greater tuberosity of the humerus. J Am Acad Orthop Surg. 2007 Oct;15(10):607-
13. Review. PubMed: 17916784.View Abstract at PubMed
2- Platzer P, Kutscha-Lissberg F, Lehr S, Vecsei V, Gaebler C. The influence of displacement on shoulder function
in patients with minimally displaced fractures of the greater tuberosity. Injury. 2005 Oct;36(10):1185-9. Epub
2005 Jun 16. PubMed PMID: 15963996. View Abstract at PubMed
3- Mattyasovszky SG, Burkhart KJ, Ahlers C, Proschek D, Dietz SO, Becker I, Müller-Haberstock S, Müller LP,
Rommens PM. Isolated fractures of the greater tuberosity of the proximal humerus: a long-term retrospective
study of 30 patients . Acta Orthop. 2011 Dec;82(6):714-20. doi: 10.3109/17453674.2011.618912. Epub 2011
Sep 6. PubMed PMID: 21895502. View Abstract at PubMed

Question 4 of 105
A 30-year-old man with diabetes sustained an acute posterior dislocation of his right
shoulder after a seizure event that required emergency department reduction. You
initially treat him with a sling for 4 weeks and then refer him for outpatient therapy.
During his therapy sessions, the patient admits to pain and instability symptoms during
range of motion exercises. Repeat examination indicates a positive posterior load-shift
test and apprehension with adduction and internal rotation of the shoulder. His CT and
MRI scans are shown in Figures 4a and 4b. What is the most appropriate next step in
treating his injury?

A B
Shoulder and Elbow Self-Assessment Examination AAOS 2014

1- Brief period of immobilization and reinitiation of therapy


2- Open posterior capsular shift with labral repair
3- Arthroscopic capsulolabral repair with subscapularis and lesser tuberosity
transfer
4- Resurfacing arthroplasty with labral repair

PREFERRED RESPONSE: 3- Arthroscopic capsulolabral repair with subscapularis


and lesser tuberosity transfer
DISCUSSION
Posterior glenohumeral dislocations are much less common than anterior
glenohumeral dislocations, with a prevalence of 1.1 per 100,000 per year. There is a
bimodal distribution with a peak in young men (2.4 men to 1 woman) and a second
peak in elderly people with a more equivalent gender ratio. Posterior dislocations most
commonly result from trauma, with the remainder of events secondary to seizure
activity. According to Robinson and associates, age younger than 40 years, dislocation
during a seizure, and a large Hill-Sachs lesion were all predictive of recurrent
instability. Concomitant injuries associated with posterior shoulder dislocations
include capsulolabral tears, fractures, and rotator cuff tears. Imaging studies in this
patient indicate a reverse Hill-Sachs lesion with a corresponding posterior labral tear.
Because of his persistent mechanical symptoms, continued immobilization and
therapy is not appropriate. An open capsular shift with labral repair alone would not
address symptoms related to an engaging Hill-Sachs lesion. Based upon the patient’s
age, activity level, and percentage of humeral head involvement, a resurfacing
arthroplasty is not recommended. Historically, the Zuckerman procedure, lesser
tuberosity transfer, has been used to address symptomatic reverse Hill-Sachs lesions
(20% to 40% humeral head involvement) associated with posterior shoulder
dislocations. Modifications of this technique such as arthroscopic transfer of the
subscapularis tendon without posterior capsulorrhaphy have proven beneficial.
RECOMMENDED READINGS
1- Rouleau DM, Hebert-Davies J. Incidence of associated injury in posterior shoulder dislocation: systematic
review of the literature. J Orthop Trauma. 2012 Apr;26(4):246-51. Review. PubMed PMID: 22183196.View
Abstract at PubMed
2- Robinson CM, Seah M, Akhtar MA. The epidemiology, risk of recurrence, and functional outcome after an
acute traumatic posterior dislocation of the shoulder. J Bone Joint Surg Am. 2011 Sep 7;93(17):1605-13.
PubMed PMID: 21915575. View Abstract at PubMed
3- Krackhardt T, Schewe B, Albrecht D, Weise K. Arthroscopic fixation of the subscapularis tendon in the reverse
Hill-Sachs lesion for traumatic unidirectional posterior dislocation of the shoulder. Arthroscopy. 2006
Feb;22(2):227.e1-227.e6. PubMed PMID: 16458812.View Abstract at PubMed
Shoulder and Elbow Self-Assessment Examination AAOS 2014

CLINICAL SITUATION FOR QUESTIONS 5 AND 6


A 40-year-old riveter who works in a manufacturing plant experiences gradual onset
of anterior shoulder pain that started 4 months ago, and he now has a constant ache in
his shoulder. His pain wakes him at night and is worsened by lifting anything at
shoulder height. He does not have any radiation of pain, and neck motion does not
aggravate his pain. He stopped doing riveting work 1 month ago, but the pain did not
improve. Examination shows normal motion and strength, a positive impingement
sign, and tenderness over the anterior greater tuberosity. His sulcus sign is negative,
and his Yergason and speed test findings are negative. He has normal scapular
mechanics.

Question 5 of 105
What is the next most appropriate step to confirm the diagnosis?
1- Radiograph
2- MRI scan
3- Subacromial injection with lidocaine
4- Ultrasound

PREFERRED RESPONSE: 3- Subacromial injection with lidocaine

Question 6 of 105
He started physical therapy while continuing light duty at work. Eight weeks later, his
pain remained unchanged. An MRI scan is shown in Figure 5. What histologic
changes are likely to be found in the
supraspinatus tendon?
1- Disorganized collagen fibers with mucoid
degeneration
2- Disorganized collagen fibers and acute
inflammatory
3- Normal tendon fibers infiltrated with
capillary proliferation
4- Normal tendon fibers infiltrated with
acute inflammatory cells

PREFERRED RESPONSE: 1- Disorganized collagen fibers with mucoid


degeneration
DISCUSSION
Shoulder and Elbow Self-Assessment Examination AAOS 2014

This patient has impingement syndrome based on the history and examination. The
best way to confirm the diagnosis is by performing a subacromial injection with
lidocaine, which is also called a Neer impingement test. If the pain is relieved, the
patient’s pain is coming from the subacromial space. An MRI scan would not confirm
the diagnosis of impingement, although it can aid in diagnosis of other causes of
anterior shoulder pain such as a rotator cuff tear. This patient has normal rotator cuff
strength, so that diagnosis is less likely. A radiograph can show acromial morphology,
which would support the diagnosis of impingement, but it does not rule out
impingement if the radiograph findings are normal. Ultrasound would not support the
diagnosis of impingement, but, like an MRI scan, it can reveal pathologies other than
impingement. The MRI scan shows a supraspinatus tendon with changes consistent
with tendinopathy, which is defined by disorganized collagen fibers with mucoid
degeneration on the microscopic level. Although there are always exceptions, most
tendinopathy occurs without inflammatory cells or capillary proliferation.
RECOMMENDED READINGS
1- Park HB, Yokota A, Gill HS, El Rassi G, McFarland EG. Diagnostic accuracy of clinical tests for the different
degrees of subacromial impingement syndrome. J Bone Joint Surg Am. 2005 Jul;87(7):1446-55. PubM:
15995110. View Abstract at PubMed
2- Khan KM, Cook JL, Bonar F, Harcourt P, Astrom M. Histopathology of common tendinopathies. Update and
implications for clinical management. Sports Med. 1999 Jun;27(6):393-408. Review. PubMed PMID:
10418074. View Abstract at PubMed
3- Lauder TD. Musculoskeletal disorders that frequently mimic radiculopathy. Phys Med Rehabil Clin N Am.
2002 Aug;13(3):469-85. Review. PubMed PMID: 12380546. View Abstract at PubMed
4- Cannon DE, Dillingham TR, Miao H, Andary MT, Pezzin LE. Musculoskeletal disorders in referrals for
suspected cervical radiculopathy. Arch Phys Med Rehabil. 2007 Oct;88(10):1256-9. PubMed PMID: 17908566.
View Abstract at PubMed

CLINICAL SITUATION FOR QUESTIONS 7 THROUGH 9


A 26-year-old man has the chief complaint of right shoulder instability. He underwent
an uncomplicated arthroscopic Bankart repair following an injury sustained while
playing high school football. His condition was stable for 7 years, but he redislocated
his shoulder in a fall 6 months ago. He describes weekly anterior instability events that
he can reduce on his own. Radiographs reveal a located glenohumeral joint, but a Hill-
Sachs lesion is noted. A CT scan reveals a 20% anteroinferior glenoid deficiency and a
Hill-Sachs lesion involving 20% of the humeral head.

Question 7 of 105
What is the best surgical option for this patient?
1- Coracoid transfer
2- Open Bankart repair
Shoulder and Elbow Self-Assessment Examination AAOS 2014

3- Revision arthroscopic Bankart repair


4- Arthroscopic remplissage procedure

PREFERRED RESPONSE: 1- Coracoid transfer

Question 8 of 105
What is the best indication to treat a Hill-Sachs lesion?
1- A lesion involving 20% of the humeral head that does not engage on
examination
2- A lesion involving 25% of the humeral head that remains located following
instability repair
3- A lesion involving 30% of the humeral head that engages on examination
4- A lesion involving 40% of the humeral head with recurrent glenohumeral
instability

PREFERRED RESPONSE: 4- A lesion involving 40% of the humeral head with


recurrent glenohumeral instability

Question 9 of 105
What is the most likely predictor of postsurgical pain following a coracoid transfer
procedure for recurrent shoulder instability?
1- Suboptimal graft placement
2- Pain before surgery
3- Progression of osteoarthritis
4- Previous surgical treatment

PREFERRED RESPONSE: 2- Pain before surgery


DISCUSSION
Recurrent glenohumeral instability with anteroinferior glenoid bone loss is best treated
with a coracoid transfer. Schmid and associates reported on 49 patients with these
lesions and recurrent instability who underwent coracoid transfer procedures. Good
and excellent outcomes were reported in 88% of patients, and there were no instances
of recurrent instability. With anteroinferior glenoid bone loss, capsular procedures—
whether open or arthroscopic—are unlikely to provide stability. Remplissage is a
procedure designed to address Hill-Sachs lesions rather than glenoid defects. The
absolute indications to treat Hill-Sachs lesions surgically include a lesion associated
with a humeral head fracture, a lesion involving more than 30% of the humeral head
with chronic instability, and reverse Hill-Sachs lesions involving more than 20% of
Shoulder and Elbow Self-Assessment Examination AAOS 2014

the articular surface with symptoms of posterior instability. Lesions involving 20% to
35% with or without engagement on examination are relative indications, as are
lesions exceeding 10% that do not remain centered in the glenoid following
arthroscopic stabilization.
In Schmid and associates’ series of coracoid transfers for recurrent instability and
anterior glenoid deficiency, patients who reported pain before surgery were 20 times
more likely to have pain after surgery that compromised the functional outcome.
Optimal graft placement correlated with better functional outcomes and less
progression of arthrosis, but not with pain. Consequently, poor graft position, arthritis
progression, and prior surgical treatment are not as consistently predictive of pain after
surgery.
RECOMMENDED READINGS
1- Schmid SL, Farshad M, Catanzaro S, Gerber C. The Latarjet procedure for the treatment of recurrence of
anterior instability of the shoulder after operative repair: a retrospective case series of forty-nine consecutive
patients. J Bone Joint Surg Am. 2012 Jun 6;94(11):e75. doi: 10.2106/JBJS.K.00380. PubMed PMID:
22637215.View Abstract at PubMed
2- Provencher MT, Frank RM, Leclere LE, Metzger PD, Ryu JJ, Bernhardson A, Romeo AA. The Hill-Sachs
lesion: diagnosis, classification, and management. J Am Acad Orthop Surg. 2012 Apr;20(4):242-52. doi:
10.5435/JAAOS-20-04-242. Review. PubMed PMID: 22474094. View Abstract at PubMed
3- Boileau P, O'Shea K, Vargas P, Pinedo M, Old J, Zumstein M. Anatomical and functional results after
arthroscopic Hill-Sachs remplissage. J Bone Joint Surg Am. 2012 Apr 4;94(7):618-26. doi:
10.2106/JBJS.K.00101. PubMed PMID: 22488618. View Abstract at PubMed

Question 10 of 105
A 45-year-old woman with diabetes has a 3-month history of left shoulder pain and
motion loss unrelated to trauma. She previously underwent treatment with
nonsteroidal anti-inflammatory medication and a home stretching program,
experiencing minimal relief of her symptoms. Examination reveals loss of passive
external rotation, abduction, and forward elevation without reduction in strength.
Radiograph findings are normal. What is the most appropriate next step?
1- MRI scan
2- Cortisone injection therapy with continued physical therapy (PT)
3- Closed manipulation under anesthesia
4- Arthroscopic release with manipulation under anesthesia

PREFERRED RESPONSE: 2- Cortisone injection therapy with continued physical


therapy (PT)
DISCUSSION
Based upon the duration of symptoms and clinical presentation, this patient would
benefit from cortisone injection therapy and continued PT. Adhesive capsulitis is most
Shoulder and Elbow Self-Assessment Examination AAOS 2014

commonly an idiopathic process that results in joint pain and loss of motion from
capsular contracture. It affects approximately 2% to 5% of the general population. The
process typically affects middle-age women. There are secondary causes such as
previous trauma and fractures as well as associated medical conditions such as
diabetes, stroke, and cardiac and thyroid disease. Debate remains as to whether there is
a genetic predisposition for the development of adhesive capsulitis despite increased
frequency noted in twin studies. Although the underlying etiology and
pathophysiology are not well understood, the consensus is that synovial inflammation
and capsular fibrosis result in pain and joint volume loss. It is hypothesized that in
patients with diabetes, an increased rate of glycosylation and cross-linking of the
shoulder capsule raises the incidence of frozen shoulder. For this patient, history
reveals a short course of symptoms that did not improve with nonsurgical modalities.
Clinically, the patient has reduced passive range of motion, particularly with external
and internal rotation and forward elevation. Radiographs are usually obtained to
exclude other causes of shoulder pain such as glenohumeral arthrosis, malignancy,
calcific tendonitis, impingement, and acromioclavicular degeneration. If pain and
stiffness persist beyond 6 months, closed manipulation may be an option.
Complications associated with this modality may include humerus fracture,
dislocation, hematoma, rotator cuff and labral tears, and brachial plexus injury. Some
surgeons advocate arthroscopic capsular release to allow for examination of
concomitant pathology and controlled release of capsular tissue, with the potential for
reduced required force when performing the manipulation portion of the procedure.
This modality may be appropriate after an initial treatment with PT. Controversy
remains as to whether posterior capsular release should be performed routinely
because studies have shown outcomes to be similar with anterior and combined
approaches. Therapy should be initiated early after intervention, with some surgeons
advocating admission to the hospital with inpatient therapy for pain management and
compliance.
RECOMMENDED READINGS
1. Neviaser AS, Neviaser RJ. Adhesive capsulitis of the shoulder. J Am Acad Orthop Surg. 2011 Sep;19(9):536-
42. Review. PubMed PMID: 21885699. View Abstract at PubMed
2. Hsu JE, Anakwenze OA, Warrender WJ, Abboud JA. Current review of adhesive capsulitis. J Shoulder Elbow
Surg. 2011 Apr;20(3):502-14. Epub 2010 Dec 16. Review. PubMed PMID: 21167743.View Abstract at PubMed
3. Le Lievre HM, Murrell GA. Long-term outcomes after arthroscopic capsular release for idiopathic adhesive
capsulitis. J Bone Joint Surg Am. 2012 Jul 3;94(13):1208-16. PubMed PMID: 22760389. View Abstract at
PubMed
4. Yian EH, Contreras R, Sodl JF. Effects of glycemic control on prevalence of diabetic frozen shoulder. J Bone
Joint Surg Am. 2012 May 16;94(10):919-23. PubMed PMID: 22617920.View Abstract at PubMed
Shoulder and Elbow Self-Assessment Examination AAOS 2014

Question 11 of 105
A 42-year-old woman has a posterior elbow dislocation. Closed reduction is
performed, and the elbow appears stable under fluoroscopic examination. Further
treatment should consist of
1- early mobilization only.
2- surgical repair or reconstruction of the lateral collateral ligament (LCL) and the
medial collateral ligament (MCL).
3- active range of motion in a hinged brace with a range of 30 degrees to 120
degrees.
4- application of a hinged external fixator with early mobilization.

PREFERRED RESPONSE: 1- early mobilization only.


DISCUSSION
This is a simple (no associated fracture) elbow dislocation. Such dislocations can be
treated with closed reduction followed by mobilization after 5 to 7 days to avoid
stiffness, provided the elbow is stable through a full arc of motion at the time of
reduction. If the elbow is unstable but has a short arc of stability, then using a hinged
brace in the stable arc may be considered. (Note: It may be necessary to splint the
elbow in pronation if the MCL is intact and the LCL is disrupted or in supination if the
LCL is intact but the MCL disrupted.) Surgical reconstruction of the LCL and MCL
may be required only if the elbow does not have a stable arc at the time of reduction. If
unstable after reconstruction, application of a hinged external fixator may be
considered.
RECOMMENDED READINGS
1- Glaser D, Armstrong A. Elbow and forearm trauma. In: Flynn J ed. Orthopaedic Knowledge Update 10.
Rosemont, IL: American Academy of Orthopaedic Surgeons; 2011:325-341.
2- Anakwe RE, Middleton SD, Jenkins PJ, McQueen MM, Court-Brown CM. Patient-reported outcomes after
simple dislocation of the elbow. J Bone Joint Surg Am. 2011 Jul 6;93(13):1220-6. doi: 10.2106/JBJS.J.00860.
PubMed PMID: 21776575.View Abstract at PubMed
3- Mehlhoff TL, Noble PC, Bennett JB, Tullos HS. Simple dislocation of the elbow in the adult. Results after
closed treatment. J Bone Joint Surg Am. 1988 Feb;70(2):244-9. PubMed PMID: 3343270. View Abstract at
PubMed

RESPONSES FOR QUESTIONS 12 THROUGH 16


1- Physical therapy and activity modification
2- Corticosteroid injection
3- Arthroscopic glenohumeral capsular release
4- Arthroscopic superior labrum anterior to posterior (SLAP) repair
5- Arthroscopic subacromial decompression and rotator cuff debridement
6- Arthroscopic subacromial decompression and rotator cuff repair
Shoulder and Elbow Self-Assessment Examination AAOS 2014

Match the treatment above with the clinical scenario below

Question 12 of 105
A 65-year-old woman has 4 months of atraumatic shoulder pain persisting despite
physical therapy and activity modification. She has normal range of motion, and an
MRI scan reveals a 10% thickness partial articular supraspinatus tear.
1- Physical therapy and activity modification
2- Corticosteroid injection
3- Arthroscopic glenohumeral capsular release
4- Arthroscopic superior labrum anterior to posterior (SLAP) repair
5- Arthroscopic subacromial decompression and rotator cuff debridement
6- Arthroscopic subacromial decompression and rotator cuff repair

PREFERRED RESPONSE: 2- Corticosteroid injection

Question 13 of 105
A 35-year-old mechanic has 6 months of shoulder pain following an axial traction
work-related injury. His pain has persisted despite extensive physical therapy and
work restrictions. A noncontrast MRI scan shows a 90% partial bursal-sided
supraspinatus tear.
1- Physical therapy and activity modification
2- Corticosteroid injection
3- Arthroscopic glenohumeral capsular release
4- Arthroscopic superior labrum anterior to posterior (SLAP) repair
5- Arthroscopic subacromial decompression and rotator cuff debridement
6- Arthroscopic subacromial decompression and rotator cuff repair

PREFERRED RESPONSE: 6- Arthroscopic subacromial decompression and rotator


cuff repair

Question 14 of 105
A 25-year-old tennis player has a type II SLAP lesion, with 4 weeks of new-onset
atraumatic shoulder pain.
1- Physical therapy and activity modification
2- Corticosteroid injection
3- Arthroscopic glenohumeral capsular release
4- Arthroscopic superior labrum anterior to posterior (SLAP) repair
5- Arthroscopic subacromial decompression and rotator cuff debridement
Shoulder and Elbow Self-Assessment Examination AAOS 2014

6- Arthroscopic subacromial decompression and rotator cuff repair

PREFERRED RESPONSE: 1- Physical therapy and activity modification

Question 15 of 105
A 49-year-old woman has 12 months of shoulder pain, global glenohumeral motion
loss, and is nonresponsive to a home stretching program and an intra-articular
glenohumeral corticosteroid injection. MRI scans reveal no full-thickness rotator cuff
tears.
1- Physical therapy and activity modification
2- Corticosteroid injection
3- Arthroscopic glenohumeral capsular release
4- Arthroscopic superior labrum anterior to posterior (SLAP) repair
5- Arthroscopic subacromial decompression and rotator cuff debridement
6- Arthroscopic subacromial decompression and rotator cuff repair

PREFERRED RESPONSE: 3- Arthroscopic glenohumeral capsular release

Question 16 of 105
A 75-year-old man has had 8 months of persistent, atraumatic shoulder pain. He had
transient improvement with physical therapy and a subacromial corticosteroid
injection. MRI scan shows a 25% partial articular supraspinatus/subscapularis tear and
significant subacromial bursal inflammation.
1- Physical therapy and activity modification
2- Corticosteroid injection
3- Arthroscopic glenohumeral capsular release
4- Arthroscopic superior labrum anterior to posterior (SLAP) repair
5- Arthroscopic subacromial decompression and rotator cuff debridement
6- Arthroscopic subacromial decompression and rotator cuff repair

PREFERRED RESPONSE: 5- Arthroscopic subacromial decompression and rotator


cuff debridement
DISCUSSION
The management of partial rotator cuff tears depends upon many factors, including
patient age, symptom onset and severity, prior treatment, physical limitation(s) based
on history and examination, and the extent of structural involvement based upon
detailed imaging (typically MRI or ultrasound). The incidence of a partial rotator cuff
tear, based on imaging with MRI or ultrasound, is high in patients ages 60 years and
Shoulder and Elbow Self-Assessment Examination AAOS 2014

older. Patients beyond age 60 with either mild or new-onset symptoms with preserved
active and passive range of motion are excellent candidates for physical therapeutic
intervention and avoidance of exacerbating activities, particularly when MRI scan or
ultrasound reveal less than 50% tendon involvement. Partial rotator cuff tears are also
common in the dominant arm of overhead athletes, and frequently respond to
nonsurgical treatment, as well. These types of partial rotator cuff tears often are seen
in combination with superior labral pathology. Rotator cuff repair usually is
recommended for patients with tears that involve more than 50% of tendon thickness
who have failed a reasonable attempt at nonsurgical management, particularly patients
who are young and have high activity demands. Partial-sided bursal tears may be more
symptomatic and respond well to surgical repair, but patients may not do as well with
subacromial decompression alone. Global loss of glenohumeral motions is consistent
with adhesive capsulitis. Such patients are initially treated with therapy that
emphasizes range of motion, usually incorporating a home exercise program. Finally,
subacromial decompression may be considered for patients with low-grade partial
articular rotator cuff tears that have failed nonsurgical management and substantially
interfere with daily and/or recreational activities.
RECOMMENDED READINGS
1- Sher JS, Uribe JW, Posada A, Murphy BJ, Zlatkin MB. Abnormal findings on magnetic resonance images of
asymptomatic shoulders. J Bone Joint Surg Am. 1995 Jan;77(1):10-5..View Abstract at PubMed
2- Weber SC. Arthroscopic debridement and acromioplasty versus mini-open repair in the treatment of significant
partial-thickness rotator cuff tears. Arthroscopy. 1999 Mar;15(2):126-31. PubMed PMID: 10210067. View
Abstract at PubMed
3- Cordasco FA, Backer M, Craig EV, Klein D, Warren RF. The partial-thickness rotator cuff tear: is
acromioplasty without repair sufficient? Am J Sports Med. 2002 Mar-Apr;30(2):257-60. PubMed PMID:
11912097.View Abstract at PubMed
4- Connor PM, Banks DM, Tyson AB, Coumas JS, D'Alessandro DF. Magnetic resonance imaging of the
asymptomatic shoulder of overhead athletes: a 5-year follow-up study. Am J Sports Med. 2003 Sep-
Oct;31(5):724-7. PubMed PMID: 12975193.View Abstract at PubMed
5- Yamaguchi K, Ditsios K, Middleton WD, Hildebolt CF, Galatz LM, Teefey SA. The demographic and
morphological features of rotator cuff disease. A comparison of asymptomatic and symptomatic shoulders. J
Bone Joint Surg Am. 2006 Aug;88(8):1699-704. PubMed PMID: 16882890.View Abstract at PubMed
6- Loeffler BJ, Brown SL, D'Alessandro DF, Fleischli JE, Connor PM. Incidence of False Positive Rotator Cuff
Pathology in MRIs of Patients with Adhesive Capsulitis. Orthopedics. 2011 May 18;34(5):362. doi:
10.3928/01477447-20110317-14. PubMed PMID: 21598899.View Abstract at PubMed
7- Pedowitz RA, Yamaguchi K, Ahmad CS, Burks RT, Flatow EL, Green A, Iannotti JP, Miller BS, Tashjian RZ,
Watters WC 3rd, Weber K, Turkelson CM, Wies JL, Anderson S, St Andre J, Boyer K, Raymond L, Sluka P,
McGowan R; American Academy of Orthopaedic Surgeons. Optimizing the management of rotator cuff
problems. J Am Acad Orthop Surg. 2011 Jun;19(6):368-79. PubMed: 21628648.View Abstract at PubMed

Question 17 of 105
The fracture seen in Figure 17 is most likely associated with injury to what
ligamentous structure?
Shoulder and Elbow Self-Assessment Examination AAOS 2014

1- Inferior glenohumeral ligament


2- Acromioclavicular (AC) ligaments
3- Coracoclavicular ligaments
4- Coracoacromial ligament

PREFERRED RESPONSE: 3- Coracoclavicular ligaments


DISCUSSION
The radiograph shows an extra-articular distal clavicle fracture lateral to the clavicular
attachment point of the coracoclavicular ligaments (conoid and trapezoid). However,
unlike a scenario featuring a typical Neer type I fracture, the interval between coracoid
and clavicle is clearly widened and there is marked fracture displacement. It is clear
that the coracoclavicular ligaments must also be torn. The inferior glenohumeral
ligament is important to glenohumeral joint stability, but has no effect on the
relationship between clavicle and scapula. The AC ligaments are thickenings of the
AC joint capsule. They have been shown to be responsible for 90% of anteroposterior
stability of the AC joint. The coracoclavicular ligaments are responsible for 77% of
stability for superior translation (as in this case). The coracoacromial ligament
connects 2 parts of the scapula (coracoids and acromion) and is part of the arch that
supports the rotator cuff.
RECOMMENDED READINGS
1- NEER CS 2nd. Fracture of the distal clavicle with detachment of the coracoclavicular ligaments in adults. J
Trauma. 1963 Mar;3:99-110. PubMed PMID: 13937900.View Abstract at PubMed
2- URIST MR. COMPLETE DISLOCATION OF THE ACROMIOCLAVICULAR JOINT. J Bone Joint Surg
Am. 1963 Dec;45:1750-3. PubMed PMID: 14083156.View Abstract at PubMed
3- Acus RW 3rd, Bell RH, Fisher DL. Proximal clavicle excision: an analysis of results. J Shoulder Elbow Surg.
1995 May-Jun;4(3):182-7. PubMed PMID: 7552675.View Abstract at PubMed

Question 18 of 105
A 36-year-old right-hand-dominant man fell from his motorcycle and sustained the
acute right upper extremity injury seen in Figure 18. At surgery, an open reduction and
internal fixation of the ulna is performed along with attempted open reduction of the
radiocapitellar joint. However, the radial head is slightly subluxed in flexion and
redislocates with elbow extension below 90 degrees. What is the most appropriate
treatment at this time?
Shoulder and Elbow Self-Assessment Examination AAOS 2014

1- Radial head resection


2- Casting in 90 degrees of flexion for 3 weeks, followed by reassessment of
elbow stability
3- Repair of the annular ligament
4- Revision open reduction and internal fixation of the ulnar fracture

PREFERRED RESPONSE: 4- Revision open reduction and internal fixation of the


ulnar fracture
DISCUSSION
This case is a variant of a type I Monteggia fracture according to the Bado
classification with a segmental ulna fracture. In some cases, the radial head
subluxation can be subtle, and missing this would lead to a poor outcome. In this case,
the anterior radial head dislocation is obvious, but the segmental nature of the ulna
fracture makes anatomic reduction difficult. The radial head usually spontaneously
reduces once the ulna is anatomically reduced, and no surgical treatment to the lateral
side is required. When this is not the case, a lateral approach and incision of the
annular ligament may be required for reduction. If an open reduction of the radial head
is unsuccessful, the problem is almost always residual malalignment of the ulna.
Therefore, casting and annular ligament repair will not improve reduction. A radial
head resection would eliminate the nonconcentric contact between radial head and
capitellum, but would not be an appropriate treatment for this young patient who has
an acute, correctable fracture deformity.
RECOMMENDED READINGS
1- Bado JL. The Monteggia lesion. Clin Orthop Relat Res. 1967 Jan-Feb;50:71-86. PubMed PMID: 6029027.
View Abstract at PubMed
2- Ring D, Jupiter JB, Waters PM. Monteggia fractures in children and adults. J Am Acad Orthop Surg. 1998 Jul-
Aug;6(4):215-24. Review. PubMed PMID: 9682084. View Abstract at PubMed

Question 19 of 105
Figure 19 is the radiograph of a 45-year-old right-hand-dominant man who has had a
2-day history of right shoulder pain, weakness, and a deformity involving the clavicle
region after a fall from a scaffold during work activities. He was previously evaluated
Shoulder and Elbow Self-Assessment Examination AAOS 2014

by his primary care physician and another orthopaedist. He has sought a second
opinion regarding his treatment options. What is the most appropriate treatment for his
injury?

1- Sling immobilization with continued observation


2- Open reduction and plate fixation
3- Open reduction, plate fixation, and application of an external bone stimulator
4- External fixation

PREFERRED RESPONSE: 2- Open reduction and plate fixation


DISCUSSION
Midshaft clavicle fractures are relatively common and account for 3% to 10% of all
adult fractures. Historical studies regarding nonsurgical treatment of displaced clavicle
fractures indicated a low nonunion rate approaching 1%. Contemporary studies,
however, suggest that the nonunion rate is much higher, reaching 15% to 20% with
corresponding loss of shoulder strength and residual deficits at 6 months from date of
injury. A recent meta-analysis by McKee and associates comparing nonsurgical
treatment with a sling and surgical treatment with plate fixation concluded that initial
fixation of displaced midshaft clavicle fractures demonstrated a positive effect on pain
reduction at 3 weeks and improved Disabilities of the Arm, Shoulder, and Hand
(DASH) scores at 6 weeks and 3 months after surgery. The choice of fixation remains
a matter of debate because both plate and pin fixation have been used to achieve
clavicle union. Intramedullary fixation of clavicle fractures historically has
demonstrated an increased risk for pin tract infections and hardware removal
attributable to local irritation from the implant. External bone stimulator use has not
proven beneficial in effecting a reduction of nonunion rates. The most common
Shoulder and Elbow Self-Assessment Examination AAOS 2014

complications noted with surgical intervention included local hardware irritation and
wound infection.
RECOMMENDED READINGS
1- McKee RC, Whelan DB, Schemitsch EH, McKee MD. Operative versus nonoperative care of displaced
midshaft clavicular fractures: a meta-analysis of randomized clinical trials. J Bone Joint Surg Am. 2012 Apr
18;94(8):675-84. Review. PubMed PMID: 22419410. View Abstract at PubMed
2- Canadian Orthopaedic Trauma Society. Nonoperative treatment compared with plate fixation of displaced
midshaft clavicular fractures. A multicenter, randomized clinical trial. J Bone Joint Surg Am. 2007 Jan;89(1):1-
10. PubMed PMID: 17200303.View Abstract at PubMed
3- Judd DB, Pallis MP, Smith E, Bottoni CR. Acute operative stabilization versus nonoperative management of
clavicle fractures. Am J Orthop (Belle Mead NJ). 2009 Jul;38(7):341-5. PubMed PMID: 19714275. View
Abstract at PubMed

Question 20 of 105
A 55-year-old woman develops posttraumatic arthritis in the elbow following a distal
humerus fracture. What is the most likely mid-term (5-10 years after surgery)
complication following semiconstrained total elbow arthroplasty (TEA)?
1- Bushing wear
2- Infection
3- Aseptic component loosening
4- Component fracture

PREFERRED RESPONSE: 1- Bushing wear


DISCUSSION
TEA has been described for posttraumatic arthritis of the elbow and typically involves
a young patient population with multiple previous operations on the affected elbow.
Morrey and Schneeberger found aseptic component loosening to be uncommon (fewer
than 10% of patients), and usually occurs more than 10 years after surgery. Prosthetic
fracture, usually of the ulnar component, is also a late-term finding. Infection is the
most common mode of early failure, but usually occurs within the first 5 years and has
an overall rate of approximately 5%. Bushing wear has been reported as the most
common cause of mechanical TEA failure in this population at intermediate-term
followup.
RECOMMENDED READINGS
1- Schneeberger AG, Adams R, Morrey BF. Semiconstrained total elbow replacement for the treatment of post-
traumatic osteoarthrosis. J Bone Joint Surg Am. 1997 Aug;79(8):1211-22. PubMed PMID: 9278082.
View Abstract at PubMed
2- Morrey BF, Schneeberger AG. Total elbow arthroplasty for posttraumatic arthrosis. Instr Course Lect.
2009;58:495-504. PubMed PMID: 19385558.View Abstract at PubMed
3- Throckmorton T, Zarkadas P, Sanchez-Sotelo J, Morrey B. Failure patterns after linked semiconstrained total
elbow arthroplasty for posttraumatic arthritis. J Bone Joint Surg Am. 2010 Jun;92(6):1432-41. doi:
10.2106/JBJS.I.00145. PubMed PMID: 20516319. View Abstract at PubMed
Shoulder and Elbow Self-Assessment Examination AAOS 2014

CLINICAL SITUATION FOR QUESTIONS 21 AND 22


A 21-year-old rugby player has had the sensation of shoulder instability while making
tackles for 3 years. Two years ago, he had an arthroscopic Bankart repair and
capsulorrhaphy that used 3 suture anchors after dislocating his shoulder while making
a tackle. This procedure required an emergency department sedated reduction. After
this dislocation, he had paresthesias in his arm and a sense of weakness. His numbness
eventually resolved. He did well after surgery until 2 weeks ago, when he again felt
his shoulder dislocate while tackling and had an emergency department reduction.

Question 21 of 105
What caused his recurrent instability?
1- The use of suture anchors in his repair
2- The physical therapy program after surgery
3- His age at the time of first surgery
4- His activity levels after surgery

PREFERRED RESPONSE: 4- His activity levels after surgery

Question 22 of 105
Numbness after his first dislocation was related to
1- intrasurgical traction on the musculocutaneous nerve.
2- residual interscalene blockade.
3- ulnar neuropathy after sling use.
4- sensory axillary nerve palsy from his dislocation.

PREFERRED RESPONSE: 4- sensory axillary nerve palsy from his dislocation.


DISCUSSION
Several studies have demonstrated the equivalency of arthroscopic Bankart repair to
open surgery, but most studies have also demonstrated increased recurrence rates in
patients who participate in collision sports such as rugby. Revision surgery with
arthroscopy is unlikely to be durable, and in patients who have large glenoid defects, a
coracoid process transfer (Latarjet or Bristow) is the preferred surgical treatment.
Arthroplasty would not be indicated in a young and active patient.
RECOMMENDED READINGS
1- Neyton L, Young A, Dawidziak B, Visona E, Hager JP, Fournier Y, Walch G. Surgical treatment of anterior
instability in rugby union players: clinical and radiographic results of the Latarjet-Patte procedure with
minimum 5-year follow-up. J Shoulder Elbow Surg. 2012 Dec;21(12):1721-7. doi: 10.1016/j.jse.2012.01.023.
Epub 2012 May 5. PMID 22565042. View Abstract at PubMed
2- Robinson CM, Shur N, Sharpe T, Ray A, Murray IR. Injuries associated with traumatic anterior glenohumeral
dislocations. J Bone Joint Surg Am. 2012 Jan 4;94(1):18-26. PMID: 22218378. View Abstract at PubMed
Shoulder and Elbow Self-Assessment Examination AAOS 2014

Question 23 of 105
Figures 23a through 23d are the radiographs and MRI scans of a 30-year-old otherwise
healthy man who sustained an anterior right shoulder dislocation while playing
baseball. He requires a closed reduction under sedation at a local emergency
department. He is placed into a shoulder immobilizer and referred to your office for
further treatment. Upon inquiry, the patient states that he previously dislocated the
shoulder twice within the last year while playing basketball. He demonstrates positive
apprehension and speed tests. What is the most appropriate next treatment step?

A B

C D
1- Brief period of immobilization with initiation of therapy
2- Arthroscopic labral repair
3- Open capsular shift
4- Coracoid transfer

PREFERRED RESPONSE: 2- Arthroscopic labral repair


DISCUSSION
By history, this patient has recurrent anterior instability. His radiographs do not
indicate significant deficiency of the anterior glenoid, which would potentially require
Shoulder and Elbow Self-Assessment Examination AAOS 2014

augmentation with a coracoid transfer to restore stability to the glenohumeral joint. An


MRI scan reveals a displaced anteroinferior labral tear (Bankart lesion) with extension
into the biceps labral complex. An open capsular shift can address capsular
redundancy, but an arthroscopic procedure will allow for examination of concomitant
pathology and the ability to address the capsulolabral tear with reduced morbidity. To
minimize this patient's redislocation risk with sports activities, an arthroscopic
capsulolabral repair involving the anteroinferior and superior labrum is the most
appropriate solution. Hantes and associates demonstrated that clinical outcomes are
similar at 2-year followup in patients with combined anteroinferior and superior labral
tears vs patients with isolated Bankart lesions when treated by arthroscopic means.
RECOMMENDED READINGS
1- Carreira DS, Mazzocca AD, Oryhon J, Brown FM, Hayden JK, Romeo AA. A prospective outcome evaluation
of arthroscopic Bankart repairs: minimum 2-year follow-up. Am J Sports Med. 2006 May;34(5):771-7. PubMed
PMID: 16627629. View Abstract at PubMed
2- Taylor DC, Arciero RA. Pathologic changes associated with shoulder dislocations. Arthroscopic and physical
examination findings in first-time, traumatic anterior dislocations. Am J Sports Med. 1997 May-Jun;25(3):306-
11. PubMed PMID: 9167808. View Abstract at PubMed
3- Hantes ME, Venouziou AI, Liantsis AK, Dailiana ZH, Malizos KN. Arthroscopic repair for chronic anterior
shoulder instability: a comparative study between patients with Bankart lesions and patients with combined
Bankart and superior labral anterior posterior lesions. Am J Sports Med. 2009 Jun;37(6):1093-8. Epub 2009 Mar
13. PubMed PMID: 19286910. View Abstract at PubMed

CLINICAL SITUATION FOR QUESTIONS 24 THROUGH 26


A 16-year-old competitive skier sustained an anterior shoulder dislocation. Her
shoulder was reduced in the emergency department and placed in a sling and swathe.
She follows up with you 2 weeks later.

Question 24 of 105
You should counsel this patient and family
1- to have immediate surgery so that she may
finish the ski season.
2- that external rotation bracing now will
prevent recurrence.
3- that even with a large bone defect (>20%),
arthroscopic surgery is successful.
4- that 2 weeks of immobilization followed by
therapy may allow her to return to finish the season.

PREFERRED RESPONSE: 4- that 2 weeks of immobilization followed by therapy


may allow her to return to finish the season.
Shoulder and Elbow Self-Assessment Examination AAOS 2014

Question 25 of 105
The family opts for nonsurgical treatment with therapy and a brace to finish her
season. Because instability symptoms continue, an MRI scan is obtained and reveals a
Bankart lesion. You recommend
1- thermal capsulorrhaphy.
2- arthroscopic Bankart repair.
3- arthroscopic Latarjet.
4- open Magnuson-Stack.

PREFERRED RESPONSE: 2- arthroscopic Bankart repair.

Question 26 of 105
The patient underwent an uneventful arthroscopic repair and did well until 1 year later
when she crashed during a race. She tore her anterior cruciate ligament (ACL) and
underwent reconstruction. Followup after her successful ACL reconstruction reveals
complaints of new shoulder pain and posterior instability from using crutches after her
ACL surgery. A new MRI scan is shown in Figure 24. What factors are most likely
associated with this patient’s recurrence?
1- Gender
2- Age
3- Sport
4- Bone loss

PREFERRED RESPONSE: 2- Age


DISCUSSION
Nonsurgical treatment for a first-time traumatic dislocation may not be effective in a
young, athletic patient; moreover, a Bankart lesion may be present in 70% of all
traumatic subluxations. A period of immobilization in external rotation may decrease
the risk for recurrence, but only if the patient is placed in a brace immediately and
complies with the treatment. Return to play may be possible after 2 weeks of
immobilization, provided the patient undergoes appropriate range of motion, cuff
strengthening, and scapular stabilization exercises. Among athletes in 1 study, 80%
were able to finish the season, but 40% experienced episodes of subluxation or
recurrent dislocation prior to the end of the season.
The rate of recurrence after arthroscopic stabilization should be lower than 10% and
may be equivalent to open repair for most patients. Bone loss remains the primary
indication for open procedures, as coracoid transfers or other bone-grafting operations
may be needed to reconstruct the glenoid if more than 20% of its surface area is
Shoulder and Elbow Self-Assessment Examination AAOS 2014

missing. Furthermore, collision athletes may fare better with open surgery than
arthroscopic options. Bone loss remains the most significant factor for recurrence
across many studies. Glenoid bone loss may be present in 20% of primary dislocations
and 70% of recurrent dislocations. Age younger than 30 has a high correlation with
recurrence. Although men may be almost twice as likely as women to have a recurrent
dislocation, age seems to be the most reliable patient-related risk factor for recurrence.
Thermal capsulorrhaphy has not proven to be effective and carries a high risk for
complication. More recent studies have found equal recurrence rates between open
and arthroscopic Bankart repair, with a greater loss of motion in patients who
underwent open repair. Longitudinal studies have demonstrated that 40% to 50% are
likely to develop arthritis after a shoulder dislocation; however, recurrent dislocation
seems to be the most important factor for early development of arthritis, while age
younger than 25 may be protective. Postcapsulorrhaphy arthropathy may be more
associated with open repairs or those that severely limit external rotation.
RECOMMENDED READINGS
1- DeAngelis NA, Busconi BD, Mozzocca AD, Arciero RA. Recurrent anterior shoulder instability. In: Galatz LM,
ed. Orthopaedic Knowledge Update Shoulder and Elbow 3. Rosemont, IL: American Academy of Orthopaedic
Surgeons; 2008:93-102.
2- Ahmed I, Ashton F, Robinson CM. Arthroscopic Bankart repair and capsular shift for recurrent anterior
shoulder instability: functional outcomes and identification of risk factors for recurrence. J Bone Joint Surg Am.
2012 Jul 18;94(14):1308-15. doi: 10.2106/JBJS.J.01983. PubMed PMID: 22810402. View Abstract at PubMed
3- Boileau P, Villalba M, Héry JY, Balg F, Ahrens P, Neyton L. Risk factors for recurrence of shoulder instability
after arthroscopic Bankart repair. J Bone Joint Surg Am. 2006 Aug;88(8):1755-63. PubMed PMID: 16882898.
View Abstract at PubMed
4- Paterson WH, Throckmorton TW, Koester M, Azar FM, Kuhn JE. Position and duration of immobilization after
primary anterior shoulder dislocation: a systematic review and meta-analysis of the literature. J Bone Joint Surg
Am. 2010 Dec 15;92(18):2924-33. doi: 10.2106/JBJS.J.00631. Review. PubMed PMID: 21159993. View
Abstract at PubMed
5- Boileau P, Fourati E, Bicknell R. Neer modification of open Bankart procedure: what are the rates of recurrent
instability, functional outcome, and arthritis? Clin Orthop Relat Res. 2012 Sep;470(9):2554-60. doi:
10.1007/s11999-012-2296-5. Epub 2012 Mar 1. PubMed PMID: 22395874. View Abstract at PubMed
6- Hovelius L, Sandström B, Saebö M. One hundred eighteen Bristow-Latarjet repairs for recurrent anterior
dislocation of the shoulder prospectively followed for fifteen years: study II-the evolution of dislocation
arthropathy. J Shoulder Elbow Surg. 2006 May-Jun;15(3):279-89. PubMed PMID: 16679226. View Abstract at
PubMed
7- Hovelius L, Saeboe M. Neer Award 2008: Arthropathy after primary anterior shoulder dislocation--223
shoulders prospectively followed up for twenty-five years. J Shoulder Elbow Surg. 2009 May-Jun;18(3):339-47.
doi: 10.1016/j.jse.2008.11.004. Epub 2009 Feb 28. PubMed PMID: 19254851. View Abstract at PubMed

Question 27 of 105
Figure 27 is the MRI scan of a 63-year-old man who has experienced 3 years of
shoulder pain. He has had 2 fluoroscopically guided corticosteroid injections that
provided him with temporary but significant relief. His primary care physician ordered
an MRI scan because of his ongoing shoulder issues. His examination shows
Shoulder and Elbow Self-Assessment Examination AAOS 2014

significant loss of motion in all planes but good motor strength. The best treatment at
this point would include

1- hyaluronic acid injection.


2- physical therapy.
3- reverse total shoulder arthroplasty.
4- anatomic total shoulder arthroplasty.

PREFERRED RESPONSE: 4- anatomic total shoulder arthroplasty.


DISCUSSION
Glenohumeral arthritis in this age group is best treated with total shoulder arthroplasty.
It provides more durable and cost-effective relief than hemiarthroplasty. Hyaluronic
acid has been demonstrated in some studies to be effective at improving pain in the
short term, but is not approved by the U.S. Food and Drug Administration for use in
the shoulder. Reverse total shoulder replacement is indicated in patients who have an
irreparable rotator cuff tear. This image clearly shows the supraspinatus is intact.
Physical therapy has not proven beneficial in the setting of end-stage glenohumeral
arthritis.
RECOMMENDED READINGS
1- Izquierdo R, Voloshin I, Edwards S, Freehill MQ, Stanwood W, Wiater JM, Watters WC 3rd, Goldberg MJ,
Keith M, Turkelson CM, Wies JL, Anderson S, Boyer K, Raymond L, Sluka P; Treatment of glenohumeral
osteoarthritis. American Academy of Orthopedic Surgeons. J Am Acad Orthop Surg. 2010 Jun;18(6):375-82.
PMID: 20511443.View Abstract at PubMed
2- Mather RC 3rd, Watters TS, Orlando LA, Bolognesi MP, Moorman CT 3rd. Cost effectiveness analysis of
hemiarthroplasty and total shoulder arthroplasty. J Shoulder Elbow Surg. 2010 Apr;19(3):325-34. PMID:
20303459.View Abstract at PubMed
3- Edwards TB, Kadakia NR, Boulahia A, Kempf JF, Boileau P, Némoz C, Walch G. A comparison of
hemiarthroplasty and total shoulder arthroplasty in the treatment of primary glenohumeral osteoarthritis: results
of a multicenter study. J Shoulder Elbow Surg. 2003 May-Jun;12(3):207-13. PMID: 12851570. View Abstract
at PubMed
4- Gartsman GM, Roddey TS, Hammerman SM. Shoulder arthroplasty with or without resurfacing of the glenoid
in patients who have osteoarthritis. J Bone Joint Surg Am. 2000 Jan;82(1):26-34. PMID: 10653081. View
Abstract at PubMed

CLINICAL SITUATION FOR QUESTIONS 28 AND 29


Shoulder and Elbow Self-Assessment Examination AAOS 2014

Figure 28 is the radiograph of a 39-year-old man who had a syncopal episode and fall.
After being cleared by the emergency department, he is referred to your office for left
shoulder pain and loss of external rotation.

Question 28 of 105
What is the most likely diagnosis?
1- Rotator cuff tear
2- Adhesive capsulitis
3- Brachial plexus
4- Posterior shoulder dislocation

PREFERRED RESPONSE: 4- Posterior shoulder dislocation

Question 29 of 105
What is the best next step?
1- Physical therapy
2- CT scan
3- Arthroscopic capsular release
4- Arthroscopic Bankart repair

PREFERRED RESPONSE: 2- CT scan


DISCUSSION
The patient has a posterior glenohumeral dislocation, as evidenced by the overlap on
the initial radiograph. While posterior dislocations are rare, they can be overlooked. A
CT scan will accurately show the lesion prior to a trip to the operating room. If a
simple closed reduction is performed acutely and the arm is stable after the reduction,
no further intervention may be needed and treatment can be successful with a 2-week
period of immobilization for defects involving less than 30% of the humeral head.
However, in this scenario, open reduction is likely and stabilization may require a
modified McLaughlin procedure or other intervention to fill in the humeral defect.
Younger male patients, those with a large humeral head defect, and those with seizure
disorder may be at highest risk for recurrence. For treatment of chronic posterior
dislocations, it may be necessary to perform shoulder arthroplasty to restore stability.
Shoulder and Elbow Self-Assessment Examination AAOS 2014

Stiffness is attributable to articular incongruity; therefore, physical therapy and


capsular release are inappropriate.
RECOMMENDED READINGS
1- Duralde XA, Fogle EF. The success of closed reduction in acute locked posterior fracture-dislocations of the
shoulder. J Shoulder Elbow Surg. 2006 Nov-Dec;15(6):701-6. Epub 2006 Oct 19. PubMed PMID: 17055305.
View Abstract at PubMed
2- Robinson CM, Aderinto J. Posterior shoulder dislocations and fracture-dislocations. J Bone Joint Surg Am.
2005 Mar;87(3):639-50. Review. PubMed PMID: 15741636.View Abstract at PubMed
3- Robinson CM, Seah M, Akhtar MA. The epidemiology, risk of recurrence, and functional outcome after an
acute traumatic posterior dislocation of the shoulder. J Bone Joint Surg Am. 2011 Sep 7;93(17):1605-13. doi:
10.2106/JBJS.J.00973. PubMed PMID: 21915575. View Abstract at PubMed
4- Sperling JW, Pring M, Antuna SA, Cofield RH. Shoulder arthroplasty for locked posterior dislocation of the
shoulder. J Shoulder Elbow Surg. 2004 Sep-Oct;13(5):522-7. PubMed PMID: 15383808. View Abstract at
PubMed

Question 30 of 105
A 40-year-old right-hand-dominant construction worker has a 3-month history of right
shoulder weakness secondary to a fall from a ladder at work. He underwent
nonsurgical treatment with anti-inflammatory medication, cortisone injections, and
therapy, with minimal relief of his symptoms. A subsequent MRI scan indicates a 1-
cm full-thickness supraspinatus tendon tear. He has been referred to your clinic for
discussion of surgical intervention. The patient's nurse case manager is concerned that
he may not be able to return to his preinjury level of activity at work, even with
surgical intervention. You tell the nurse case manager that, on average, the patient will
1- be at increased risk for infection compared to patients without a Worker’s
Compensation claim.
2- have significant functional improvement after rotator cuff repair that is less
robust than that of patients without a Worker’s Compensation claim.
3- have pain relief that is equivalent to that of patients without a Worker’s
Compensation claim.
4- return to work without restrictions within a 3-month time frame.

PREFERRED RESPONSE: 2- have significant functional improvement after rotator


cuff repair that is less robust than that of patients without a Worker’s Compensation
claim.
DISCUSSION
Many factors have been associated with less-than-favorable outcomes following
rotator cuff repair such as tear size, age at time of intervention, gender, fatty
degeneration of rotator cuff musculature, and Worker’s Compensation status. Henn
and associates performed a prospective study regarding rotator cuff repairs in a cohort
Shoulder and Elbow Self-Assessment Examination AAOS 2014

of 125 patients to assess the factors that may affect outcome as measured with the
Simple Shoulder Test (SST), Disabilities of the Arm, Shoulder, and Hand (DASH),
Short Form-36 (SF-36), and Visual Analog Pain Scale (VAS). When confounding
factors were controlled, Worker’s Compensation status was an independent predictor
of poorer DASH scores. With the use of historical controls, Bhatia and associates
concluded that the vast majority (89%) of workers who underwent an arthroscopic
rotator cuff repair returned to their preoperative level of work at a mean time of 7.6
months. There was a trend toward decreased return to full duty with increased work
demands before surgery (light, medium, and heavy duty), but this result did reach
statistical significance. Alcohol consumption (more than 6 drinks per week) was the
only factor to demonstrate an association with postoperative restricted work duty and
increased rotator cuff repair failure. There is no evidence to support increased
infection rates for rotator cuff repair in Worker’s Compensation patients.
RECOMMENDED READINGS
1- Bhatia S, Piasecki DP, Nho SJ, Romeo AA, Cole BJ, Nicholson GP, Boniquit N, Verma NN. Early return to
work in workers' compensation patients after arthroscopic full-thickness rotator cuff repair. Arthroscopy. 2010
Aug;26(8):1027-34. Epub 2010 Jun 3. PubMed PMID: 20678699.View Abstract at PubMed
2- Henn RF 3rd, Kang L, Tashjian RZ, Green A. Patients with workers' compensation claims have worse outcomes
after rotator cuff repair. J Bone Joint Surg Am. 2008 Oct;90(10):2105-13. PubMed PMID: 18829907. View
Abstract at PubMed

Question 31 of 105
A 75-year-old man sustains an anterior dislocation of his reverse total shoulder
arthroplasty. What activity places the arm in the position most commonly associated
with reverse total shoulder dislocation?
1- Scratching the opposite shoulder
2- Pushing off an ipsilateral chair armrest to assist in standing up
3- Tying shoelaces on the contralateral foot
4- Brushing hair

PREFERRED RESPONSE: 2- Pushing off an ipsilateral chair armrest to assist in


standing up
DISCUSSION
Proper soft-tissue tension is critical to prevent instability of a reverse total shoulder
implanted with the deltopectoral approach; dislocation of the prosthesis is exceedingly
rare if the superior approach is employed. The arm position implicated in reverse total
shoulder instability is extension, adduction, and internal rotation, such as pushing out
of a chair. The other positions described do not involve extension of the shoulder.
RECOMMENDED READINGS
Shoulder and Elbow Self-Assessment Examination AAOS 2014

1- Gerber C, Pennington SD, Nyffeler RW. Reverse total shoulder arthroplasty. J Am Acad Orthop Surg. 2009
May;17(5):284-95. Review. PubMed PMID: Cheung E, Willis M, Walker M, Clark R, Frankle MA.
Complications in reverse total shoulder arthroplasty. J Am Acad Orthop Surg. 2011 Jul;19(7):439-49. Review.
PubMed PMID: 21724923.19411640.View Abstract at PubMed
2- Cheung E, Willis M, Walker M, Clark R, Frankle MA. Complications in reverse total shoulder arthroplasty. J
Am Acad Orthop Surg. 2011 Jul;19(7):439-49. Review. PubMed PMID: 21724923.View Abstract at PubMed
3- Walch G, Wall B, Mottier F: Complication and revision of the reverse prosthesis: A multicenter study of 457
cases. In: Walch G, Boileau P, Mole P, Favard L, Levigne C, Sirveaux f, eds. Reverse Shoulder Arthroplasty:
Clinical Results, Complications, Revision. Montpellier, France: Sauramps Médical; 2006:335-352.

Question 32 of 105
When performing an ulnar nerve decompression at the elbow, the surgeon must be
aware of the
1- median nerve as it crosses the surgical field 6 cm proximal to the medial
epicondyle.
2- medial antebrachial cutaneous nerve as it crosses the field 3 cm distal to the
medial epicondyle.
3- anterior antebrachial cutaneous nerve as it crosses the field at the medial
epicondyle.
4- posterior antebrachial cutaneous nerve that crosses the field 2 cm distal to the
medial epicondyle.

PREFERRED RESPONSE: 2- medial antebrachial cutaneous nerve as it crosses the


field 3 cm distal to the medial epicondyle.
DISCUSSION
The medial antebrachial cutaneous and medial brachial cutaneous are nerves that can
be injured during ulnar nerve decompression at the elbow. The medial antebrachial
cutaneous nerve crosses the surgical field at an average of 3.1 cm distal to the medial
epicondyle. The medial brachial cutaneous nerve crosses the field 7 cm proximal to
the medial epicondyle and arborizes into 2 to 3 terminal branches. Because the
surgical approach involves dissection on the medial side, the posterior antebrachial
cutaneous nerve is distant from the exposure. Although the median nerve potentially
can be located in the deep dissection of a submuscular transposition, it is considered
distant to an in situ decompression.
RECOMMENDED READINGS
1- Lowe JB 3rd, Maggi SP, Mackinnon SE. The position of crossing branches of the medial antebrachial cutaneous
nerve during cubital tunnel surgery in humans. Plast Reconstr Surg. 2004 Sep 1;114(3):692-6. PubMed PMID:
15318047.View Abstract at PubMed
2- Chowdhry S, Elston JB, Lefkowitz T, Wilhelmi BJ. Avoiding the medial brachial cutaneous nerve in
brachioplasty: an anatomical study. Eplasty. 2010 Jan 29;10:e16. PubMed PMID: 20165546.View Abstract at
PubMed
Shoulder and Elbow Self-Assessment Examination AAOS 2014

Question 33 of 105
Figure 33 is the radiograph of a 27-year-old bicyclist who crashes. He has an isolated
and closed injury. He is neurovascularly intact in the upper extremity. The lateral
fragment is displaced inferiorly by

1- gravity.
2- the trapezius.
3- the biceps.
4- the pectoralis minor.

PREFERRED RESPONSE: 1- gravity.


DISCUSSION
Open reduction and internal fixation with a plate and screw construct have been
demonstrated to reduce nonunion rate and improve outcomes compared to sling
immobilization for displaced clavicle fractures. Neurovascular injury and infection
risk increase, however, with surgery. In the upright position, the weight of the
extremity inferiorly displaces the lateral segment.
RECOMMENDED READINGS
1- Altamimi SA, McKee MD; Canadian Orthopaedic Trauma Society. Nonoperative treatment compared with
plate fixation of displaced midshaft clavicular fractures. Surgical technique. J Bone Joint Surg Am. 2008 Mar;90
Suppl 2 Pt 1:1-8. PMID: 18310682.View Abstract at PubMed
2- Canadian Orthopaedic Trauma Society. Nonoperative treatment compared with plate fixation of displaced
midshaft clavicular fractures. A multicenter, randomized clinical trial. J Bone Joint Surg Am. 2007 Jan;89(1):1-
10. PMID: 17200303.View Abstract at PubMed
3- Darouiche RO. Treatment of infections associated with surgical implants. N Engl J Med. 2004 Apr
1;350(14):1422-9. Review. PubMed PMID: 15070792.View Abstract at PubMed

Question 34 of 105
Placement of the most distal interlocking screw seen in
the Figures 34a and 34b radiographs most likely resulted
in what motor weakness?

1- Elbow flexion
2- Thumb interphalangeal (IP) extension
3- Index proximal IP flexion
4- Index metacarpophalangeal (MCP) abduction
Shoulder and Elbow Self-Assessment Examination AAOS 2014

PREFERRED RESPONSE: 3- Index proximal IP flexion


DISCUSSION
The most distal locking screw in this intramedullary nail construct was placed from
anterior to posterior, passing through the distal portion of the biceps and brachialis
muscle bellies. The median nerve, along with the brachial artery, is at risk as it lies
between these 2 muscles. Malrotation of the nail, producing a more anteromedial
starting point for the screw, can lead to a path that intersects with the nerve. Blunt
dissection and soft-tissue protection is warranted with this screw placement. Median
nerve injury would affect innervations of the flexor digitorum superficialis and
profundus to the index finger (among other motors). Although the dissection violates
the muscle belly of these 2 elbow flexors, measurable weakness is not typically seen.
The radial nerve has already provided function to triceps (elbow extension) proximal
to this level and lies sufficiently lateral to be more of a concern with a lateral screw
placement (thumb IP extension). The ulnar nerve (index MCP abduction) is further
medial at this level and would be at considerably lower risk than the median.
RECOMMENDED READINGS
1- Morrey BF. Anatomy of the elbow joint. In: Morrey BF, Sanchez-Sotelo J, eds. The Elbow and Its Disorders.
Philadelphia, PA: WB Saunders; 2009:11-38.
2- Bohsali KI, Wirth MA. Fractures of the proximal humerus. In: Rockwood CA, Matsen FA, Wirth MA, Lippitt
SB, eds. The Shoulder. Philadelphia, PA: WB Saunders; 2009:315-319.

Question 35 of 105
One week after closed reduction of a primary anterior shoulder dislocation, a 25-year-
old athlete should be counseled that
1- recurrence rate is reduced with 4 weeks of immobilization instead of 2 weeks of
immobilization.
2- age at the time of injury is the most consistent risk factor for recurrent
instability.
3- a majority of patients in this age group will elect to have surgery for recurrent
instability.
4- after an in-season return to sports, his likelihood of choosing surgery after the
season is 25%.

PREFERRED RESPONSE: 2- age at the time of injury is the most consistent risk
factor for recurrent instability.
DISCUSSION
In a study by Sachs and associates, age younger than 25 years at the time of
presentation was found to be the strongest predictor of recurrent instability. In this age
Shoulder and Elbow Self-Assessment Examination AAOS 2014

group (20- to 29-year-olds), only 14% elected to proceed with surgery. After an in-
season return to sports, about 50% of patients in this same study chose to proceed with
surgery after completing the season. Immobilization in a sling for longer than 2 weeks
has no effect on future instability.
RECOMMENDED READINGS
1- Sachs RA, Lin D, Stone ML, Paxton E, Kuney M. Can the need for future surgery for acute traumatic anterior
shoulder dislocation be predicted? J Bone Joint Surg Am. 2007 Aug;89(8):1665-74. PubMed PMID: 17671003.
View Abstract at PubMed
2- Paterson WH, Throckmorton TW, Koester M, Azar FM, Kuhn JE. Position and duration of immobilization after
primary anterior shoulder dislocation: a systematic review and meta-analysis of the literature. J Bone Joint Surg
Am. 2010 Dec 15;92(18):2924-33. doi: 10.2106/JBJS.J.00631. Review. PubMed PMID: 21159993. View
Abstract at PubMed

CLINICAL SITUATION FOR QUESTIONS 36 THROUGH 39


A 65-year-old man experienced 6 years of worsening shoulder pain. Examination
demonstrates stiffness and crepitus with range of motion, but full rotator cuff strength
in all planes. Radiographs show advanced shoulder osteoarthritis, and an MRI scan
ordered by the patient's primary care physician shows an intact rotator cuff.

Question 36 of 105
What is the most likely glenoid wear pattern seen in glenohumeral osteoarthritis with
an external rotation deficit?
1- Posterior wear
2- Anterior wear
3- Central wear
4- Superior wear

PREFERRED RESPONSE: 1- Posterior wear

Question 37 of 105
What surgical treatment is most likely to result in long-term pain relief and functional
improvement?
1- Hemiarthroplasty
2- Hemiarthroplasty with meniscal interposition
3- Total shoulder arthroplasty (TSA)
4- Reverse TSA

PREFERRED RESPONSE: 3- Total shoulder arthroplasty (TSA)


Shoulder and Elbow Self-Assessment Examination AAOS 2014

Question 38 of 105
What risk factor is most predictive of deep infection following TSA?
1- Posttraumatic arthritis
2- Male gender
3- Body mass index higher than 30
4- Diabetes

PREFERRED RESPONSE: 2- Male gender

Question 39 of 105
At what point of glenoid retroversion is there risk for component perforation of the
glenoid vault with traditional high side reaming and standard component
implantation?
1- 5 degrees
2- 10 degrees
3- 15 degrees
4- 20 degrees

PREFERRED RESPONSE: 4- 20 degrees


DISCUSSION
Posterior glenoid wear is the most common pattern seen in typical glenohumeral
arthritis. Central wear can also be seen, but it is less common and anterior wear is least
common. TSA is the gold standard for surgical treatment of glenohumeral
osteoarthritis. Multiple comparative studies between hemiarthroplasty and total
shoulder arthroplasty demonstrate advantages of TSA regarding pain relief and most
functional parameters. Shoulder hemiarthroplasty with meniscal interposition has been
described for young patients with glenohumeral arthritis, but outcomes at
intermediate-term followup have been inferior to those of TSA. Reverse TSA is
contraindicated in patients with an intact rotator cuff. Proprionobacterium acnes and
Staphylococcus species are the most common pathogens causing deep infection after
TSA. In a recent long-term follow-up study of total shoulder infections, male gender
and younger patient age were significantly associated with a higher infection risk.
Other comorbidities and indications for TSA were not predictive. Posterior glenoid
wear results in increased retroversion and erosion of the bony vault, which can
compromise component fixation. Iannotti and associates reported on 13 patients with
varying degrees of glenoid deformity. At 20 degrees of retroversion, optimal glenoid
component placement after eccentric reaming was associated with glenoid vault
perforation.
Shoulder and Elbow Self-Assessment Examination AAOS 2014

RECOMMENDED READINGS
1- Matsen FA III, Rockwood CA Jr, Wirth MA, Lippitt SB, Parsons M. Glenohumeral arthritis and its
management. In: Rockwood CA Jr, Matsen FA III, Wirth MA, Lippitt SB, eds. The Shoulder. Vol 2. 3rd ed.
Philadelphia, PA: Saunders; 2004:879-1000.
2- Lee BK, Vaishnav S, Rick Hatch GF 3rd, Itamura JM. Biologic resurfacing of the glenoid with meniscal
allograft: long-term results with minimum 2-year follow-up. J Shoulder Elbow Surg. 2012 Aug 25. [Epub ahead
of print] PubMed PMID: 22929583.View Abstract at PubMed
3- Bryant D, Litchfield R, Sandow M, Gartsman GM, Guyatt G, Kirkley A. A comparison of pain, strength, range
of motion, and functional outcomes after hemiarthroplasty and total shoulder arthroplasty in patients with
osteoarthritis of the shoulder. A systematic review and meta-analysis. J Bone Joint Surg Am. 2005
Sep;87(9):1947-56. Review. PubMed PMID: 16140808.View Abstract at PubMed
4- Singh JA, Sperling JW, Schleck C, Harmsen WS, Cofield RH. Periprosthetic infections after total shoulder
arthroplasty: a 33-year perspective. J Shoulder Elbow Surg. 2012 Nov;21(11):1534-41. doi:
10.1016/j.jse.2012.01.006. Epub 2012 Apr 18. PubMed PMID: 22516570.View Abstract at PubMed
5- Iannotti JP, Greeson C, Downing D, Sabesan V, Bryan JA. Effect of glenoid deformity on glenoid component
placement in primary shoulder arthroplasty. J Shoulder Elbow Surg. 2012 Jan;21(1):48-55. doi:
10.1016/j.jse.2011.02.011. Epub 2011 May 20. PubMed PMID: 21600787.View Abstract at PubMed

Question 40 of 105
A 75-year-old woman with rheumatoid arthritis and a long history of oral
corticosteroid use sustains a comminuted intra-articular distal humerus fracture. What
is the best surgical option?
1- Open reduction internal fixation (ORIF) with parallel plates
2- ORIF with orthogonal plates and iliac crest bone grafting
3- Total elbow arthroplasty (TEA)
4- Closed reduction and percutaneous pinning

PREFERRED RESPONSE: 3- Total elbow arthroplasty (TEA)


DISCUSSION
TEA is the best surgical option. McKee and associates published a multicenter
randomized controlled trial comparing ORIF to TEA in elderly patients. TEA resulted
in better 2-year clinical functional scores and more predictable outcomes compared to
ORIF. TEA was also likely to result in a lower resurgical rate; one-quarter of patients
with fractures randomized to ORIF could not achieve stable fixation. Further, Frankle
and associates reported a comparative study of TEA vs ORIF in 24 elderly women.
TEA outcomes were again superior to ORIF at a minimum of 2 years of followup.
TEA was especially useful in patients with comorbidities that compromise bone stock,
including osteoporosis and oral corticosteroid use. Closed reduction and percutaneous
pinning studies have not been published on the adult population.
RECOMMENDED READINGS
1- McKee MD, Veillette CJ, Hall JA, Schemitsch EH, Wild LM, McCormack R, Perey B, Goetz T, Zomar M,
Moon K, Mandel S, Petit S, Guy P, Leung I. A multicenter, prospective, randomized, controlled trial of open
Shoulder and Elbow Self-Assessment Examination AAOS 2014

reduction--internal fixation versus total elbow arthroplasty for displaced intra-articular distal humeral fractures
in elderly patients. J Shoulder Elbow Surg. 2009 Jan-Feb;18(1):3-12. doi: 10.1016/j.jse.2008.06.005. Epub 2008
Sep 26. PubMed PMID: 18823799.View Abstract at PubMed
2- Frankle MA, Herscovici D Jr, DiPasquale TG, Vasey MB, Sanders RW. A comparison of open reduction and
internal fixation and primary total elbow arthroplasty in the treatment of intraarticular distal humerus fractures
in women older than age 65. J Orthop Trauma. 2003 Aug;17(7):473-80. PubMed PMID: 12902784.View
Abstract at PubMed

Question 41 of 105
A 67-year-old man with right shoulder osteoarthritis remains symptomatic despite a
course of nonsurgical treatment. A CT scan of the shoulder before surgery shows
eccentric posterior glenoid wear with 10 degrees of retroversion. What is the
appropriate treatment of this glenoid bone loss?
1- Implantation of the glenoid component in 10 degrees of retroversion
2- Hemiarthroplasty
3- Eccentric reaming of glenoid
4- Posterior glenoid bone graft

PREFERRED RESPONSE: 3- Eccentric reaming of glenoid


DISCUSSION
Total shoulder arthroplasty (TSA) is superior to hemiarthroplasty for primary
osteoarthritis. The most common complication of TSA is glenoid loosening and
malposition, which are common causes of glenoid failure. Glenoid malposition
decreases the glenohumeral contact area and subsequently increases contact pressures.
Altering the stem version to accommodate glenoid retroversion does not appropriately
address soft-tissue balancing. A retroversion of less than 12 degrees to 15 degrees can
be corrected with eccentric reaming without excessively compromising glenoid bone
stock and risking glenoid vault penetration by the glenoid component. Posterior
glenoid bone grafting may be considered for glenoid retroversion exceeding 15
degrees.
RECOMMENDED READINGS
1- Levine WN, Fischer CR, Nguyen D, Flatow EL, Ahmad CS, Bigliani LU. Long-Term Follow-up of Shoulder
Hemiarthroplasty for Glenohumeral Osteoarthritis. J Bone Joint Surg Am. 2012 Nov 21;94(22):e1641-7. doi:
10.2106/JBJS.K.00603. PubMed PMID: 23172331.View Abstract at PubMed
2- Edwards TB, Kadakia NR, Boulahia A, Kempf JF, Boileau P, Némoz C, Walch G. A comparison of
hemiarthroplasty and total shoulder arthroplasty in the treatment of primary glenohumeral osteoarthritis: results
of a multicenter study. J Shoulder Elbow Surg. 2003 May-Jun;12(3):207-13. PubMed PMID: 12851570.View
Abstract at PubMed
3- Sears BW, Johnston PS, Ramsey ML, Williams GR. Glenoid bone loss in primary total shoulder arthroplasty:
evaluation and management. J Am Acad Orthop Surg. 2012 Sep;20(9):604-13. doi: 10.5435/JAAOS-20-09-604.
Review. PubMed PMID: 22941803.View Abstract at PubMed
Shoulder and Elbow Self-Assessment Examination AAOS 2014

4- Shapiro TA, McGarry MH, Gupta R, Lee YS, Lee TQ. Biomechanical effects of glenoid retroversion in total
shoulder arthroplasty. J Shoulder Elbow Surg. 2007 May-Jun;16(3 Suppl):S90-5. Epub 2006 Dec 12. PubMed
PMID: 17169588.View Abstract at PubMed
5- Nowak DD, Bahu MJ, Gardner TR, Dyrszka MD, Levine WN, Bigliani LU, Ahmad CS. Simulation of surgical
glenoid resurfacing using three-dimensional computed tomography of the arthritic glenohumeral joint: the
amount of glenoid retroversion that can be corrected. J Shoulder Elbow Surg. 2009 Sep-Oct;18(5):680-8. doi:
10.1016/j.jse.2009.03.019. Epub 2009 May 31. PubMed PMID: 19487133.View Abstract at PubMed

Question 42 of 105
Figure 42 is the MRI scan of a 52-year-old active man who fell from a ladder 6 weeks
ago and sustained an isolated glenohumeral dislocation that was reduced in the
emergency department. He wore his sling for about 2 weeks and arrived at your clinic
today after referral by his primary care doctor. Examination reveals sensation intact
throughout his hand, forearm, and shoulder girdle. Belly press examination findings
are normal, but painful. He has tenderness to palpation on the anterior shoulder and a
painful speed test. Rotator cuff repair associated with tenotomy of the indicated
structure will result in what condition when compared to tenodesis of the same
structure?

1- Decreased strength in forward


elevation
2- Increased external rotation
3- Cosmetic deformity
4- Earlier fatigability with pronation

PREFERRED RESPONSE: 3- Cosmetic deformity


DISCUSSION
Patients with subluxation of the biceps tendon and full-thickness tears of the
subscapularis require treatment of the biceps tendon. Studies have shown there is
increased likelihood for cosmetic “Popeye” deformity after tenotomy when compared
to tenodesis, but there is little or no functional deficit associated with tenotomy. In
other studies, there have been findings of supination strength reduction in patients with
tenotomy compared to those with tenodesis.
RECOMMENDED READINGS
1- Hsu AR, Ghodadra NS, Provencher MT, Lewis PB, Bach BR. Biceps tenotomy versus tenodesis: a review of
clinical outcomes and biomechanical results. J Shoulder Elbow Surg. 2011 Mar;20(2):326-32. Epub 2010 Nov
4. Review. PMID: 21051241.View Abstract at PubMed
Shoulder and Elbow Self-Assessment Examination AAOS 2014

2- Slenker NR, Lawson K, Ciccotti MG, Dodson CC, Cohen SB. Biceps tenotomy versus tenodesis: clinical
outcomes. Arthroscopy. 2012 Apr;28(4):576-82. Epub 2012 Jan 28. Review. PMID: 22284407.View Abstract at
PubMed

Question 43 of 105
A complication associated with using the Morrey approach (triceps reflecting) to
implant a semiconstrained total elbow arthroplasty is
1- loss of elbow extensor power.
2- implant dislocation.
3- implant malposition.
4- development of heterotopic ossification.

PREFERRED RESPONSE: 1- loss of elbow extensor power.


DISCUSSION
Numerous approaches can be used to implant a total elbow arthroplasty. The Morrey
approach identifies, transposes, and protects the ulnar nerve, and then subperiosteally
reflects the triceps off the ulna. The sleeve of tissue is very thin distally, and the
triceps need to be meticulously repaired at the time of closure. Implant dislocation and
malposition are less likely with an extensile approach, and dislocation is unlikely with
a semiconstrained implant. The development of heterotopic ossification is unrelated to
the surgical approach used for elbow arthroplasty.
RECOMMENDED READINGS
1- Kim JM, Mudgal CS, Konopka JF, Jupiter JB. Complications of total elbow arthroplasty. J Am Acad
Orthop Surg. 2011 Jun;19(6):328-39. Review. PubMed PMID: 21628644.View Abstract at PubMed
2- Lee DH. Linked total elbow arthroplasty. Hand Clin. 2011 May;27(2):199-213, vi. PubMed PMID:
21501791.View Abstract at PubMed
3- Voloshin I, Schippert DW, Kakar S, Kaye EK, Morrey BF. Complications of total elbow replacement:
a systematic review. J Shoulder Elbow Surg. 2011 Jan;20(1):158-68. Review. PubMed PMID:
21134667.View Abstract at PubMed

CLINICAL SITUATION FOR QUESTIONS 44 AND 45


A 19-year-old hockey player returns home from college over holiday break and
experiences multiple recurrent dislocations only 1 year after an arthroscopic
stabilization.

Question 44 of 105
What is the preferred test to evaluate this patient?
1- Electromyography
Shoulder and Elbow Self-Assessment Examination AAOS 2014

2- MRI scan with intravenous contrast


3- Bone scan
4- CT arthrogram

PREFERRED RESPONSE: 4- CT arthrogram

Question 45 of 105
The treating physician opted to perform a Latarjet coracoid transfer. What is the
primary mechanism of stability?
1- Capsular reinforcement by the coracoacromial ligament
2- Dynamic sling created by the conjoint tendon
3- Increased glenoid depth
4- Subscapularis tenodesis

PREFERRED RESPONSE: 3- Increased glenoid depth


DISCUSSION
Because bone loss is likely the biggest risk factor for this patient’s recurrence, a CT
arthrogram will provide the most accurate representation of the defect. An arthrogram
enhances the ability to evaluate the capsule and ligamentous attachments that may
have a role in recurrent instability. Collision athletes may have a lower recurrence rate
with open surgery. The Latarjet coracoid transfer achieves its primary stability through
the increased glenoid surface area that the bone block provides.
RECOMMENDED READINGS
1- Schmid SL, Farshad M, Catanzaro S, Gerber C. The Latarjet procedure for the treatment of recurrence of
anterior instability of the shoulder after operative repair: a retrospective case series of forty-nine consecutive
patients. J Bone Joint Surg Am. 2012 Jun 6;94(11):e75. doi: 10.2106/JBJS.K.00380. PubMed PMID: 22637215.
View Abstract at PubMed
2- Allain J, Goutallier D, Glorion C. Long-term results of the Latarjet procedure for the treatment of anterior
instability of the shoulder. J Bone Joint Surg Am. 1998 Jun;80(6):841-52. PubMed PMID: 9655102. View
Abstract at PubMed
3- Ghodadra N, Gupta A, Romeo AA, Bach BR Jr, Verma N, Shewman E, Goldstein J, Provencher MT.
Normalization of glenohumeral articular contact pressures after Latarjet or iliac crest bone-grafting. J Bone Joint
Surg Am. 2010 Jun;92(6):1478-89. doi: 10.2106/JBJS.I.00220. PubMed PMID: 20516324. View Abstract at
PubMed
4- Yamamoto N, Muraki T, Sperling JW, Steinmann SP, Cofield RH, Itoi E, An KN. Stabilizing mechanism in
bone-grafting of a large glenoid defect. J Bone Joint Surg Am. 2010 Sep 1;92(11):2059-66. doi:
10.2106/JBJS.I.00261. PubMed PMID: 20810855. View Abstract at PubMed
5- Thomazeau H, Courage O, Barth J, Pélégri C, Charousset C, Lespagnol F, Nourissat G, Audebert S, Guillo S,
Toussaint B, Lafosse L, Bradel J, Veillard D, Boileau P; French Arthroscopy Society. Can we improve the
indication for Bankart arthroscopic repair? A preliminary clinical study using the ISIS score. Orthop Traumatol
Surg Res. 2010 Dec;96(8 Suppl):S77-83. doi: 10.1016/j.otsr.2010.09.007. Epub 2010 Oct 28. PubMed PMID:
21035419. View Abstract at PubMed
Shoulder and Elbow Self-Assessment Examination AAOS 2014

Question 46 of 105
A 45-year-old woman has a 3-month history of left shoulder pain. She has tried 2
months of physical therapy focused on rotator cuff strengthening without experiencing
relief. A subacromial corticosteroid injection fails to provide lasting relief.
Examination reveals no atrophy or winging. She has anterior and posterior shoulder
tenderness, full symmetric forward elevation and abduction, and pain with maximal
passive forward elevation. She has pain with internal rotation in 90 degrees of forward
elevation. She has an increased distance between the antecubital fossa and coracoid
process with cross chest adduction compared to the contralateral side. No weakness is
appreciated. Radiographs reveal a type II acromion. What is the best next step?
1- Posterior capsular stretching
2- Arthroscopic subacromial decompression
3- Diagnostic acromioclavicular (AC) joint injection
4- MRI scan

PREFERRED RESPONSE: 1- Posterior capsular stretching


DISCUSSION
This patient’s examination is consistent with posterior capsular tightness, which can
mimic impingement. Four weeks of posterior capsular stretches will likely improve
motion and pain. Surgical treatment should be considered only after failure of a
dedicated stretching program. An AC joint injection would help differentiate this
condition from AC joint arthritis, but in light of the radiographic findings, arthritis is
unlikely. An MRI scan likely will not change the initial treatment at this point.
RECOMMENDED READINGS
1- Bach HG, Goldberg BA. Posterior capsular contracture of the shoulder. J Am Acad Orthop Surg. 2006
May;14(5):265-77. Review. PubMed PMID: 16675620.View Abstract at PubMed
2- Papadonikolakis A, McKenna M, Warme W, Martin BI, Matsen FA 3rd. Published evidence relevant to the
diagnosis of impingement syndrome of the shoulder. J Bone Joint Surg Am. 2011 Oct 5;93(19):1827-32. doi:
10.2106/JBJS.J.01748. Review. PubMed PMID: 22005869. View Abstract at PubMed

CLINICAL SITUATION FOR QUESTIONS 47 THROUGH 49


A 13-year-old pitcher reports the immediate onset of medial elbow pain after throwing
a pitch. Upon examination, the patient is tender to palpation at the medial epicondyle
and has pain and instability with valgus testing of the elbow.

Question 47 of 105
Which is the most appropriate diagnostic test?
1- MRI arthrogram
Shoulder and Elbow Self-Assessment Examination AAOS 2014

2- CT scan with 3-dimensional reconstructions


3- Plain radiographs of both elbows
4- Ultrasound

PREFERRED RESPONSE: 3- Plain radiographs of both elbows

Question 48 of 105
Why was your response for question 47 the most appropriate test for this patient?
1- To evaluate for apophyseal injury
2- To evaluate for osteochondral defect
3- To evaluate for hematoma
4- To evaluate for valgus overload injury

PREFERRED RESPONSE: 1- To evaluate for apophyseal injury

Question 49 of 105
If the patient were a college pitcher with a similar presentation and examination, what
structure would most likely be injured?
1- Ulnar collateral ligament
2- Pronator teres
3- Ligament of Struthers
4- Lateral collateral ligament

PREFERRED RESPONSE: 1- Ulnar collateral ligament


DISCUSSION
The patient has an acute avulsion fracture of the medial epicondyle, which can occur
in response to the valgus load placed on the elbow while throwing. Diagnosis is
confirmed by radiograph, with comparison views of the uninjured elbow to evaluate
for physeal closure vs injury. In older pitchers, the ulnar collateral ligament fails rather
than the bone of the medial epicondyle. Advanced imaging may be necessary to
confirm the diagnosis of an ulnar collateral ligament injury and/or bony injury.
RECOMMENDED READINGS
1- Osbahr DC, Chalmers PN, Frank JS, Williams RJ 3rd, Widmann RF, Green DW. Acute, avulsion fractures of
the medial epicondyle while throwing in youth baseball players: a variant of Little League elbow. J Shoulder
Elbow Surg. 2010 Oct;19(7):951-7. doi: 10.1016/j.jse.2010.04.038. Epub 2010 Aug 5. PubMed PMID:
20688542.View Abstract at PubMed
2- Gottschalk HP, Eisner E, Hosalkar HS. Medial epicondyle fractures in the pediatric population. J Am Acad
Orthop Surg. 2012 Apr;20(4):223-32. doi: 10.5435/JAAOS-20-04-223. Review. PubMed PMID: 22474092.
View Abstract at PubMed
Shoulder and Elbow Self-Assessment Examination AAOS 2014

Question 50 of 105
A 65-year-old woman has electrodiagnostic findings of ulnar nerve entrapment at the
elbow. You counsel the patient that
1- medial epicondylectomy and submuscular transposition is the preferred
treatment.
2- arthroscopic decompression is associated with lower complication rates
compared to open treatment.
3- simple decompression can be as effective as ulnar nerve transposition.
4- duration of symptoms is the most important predictor of outcome.

PREFERRED RESPONSE: 3- simple decompression can be as effective as ulnar


nerve transposition.
DISCUSSION
Multiple studies have demonstrated that simple ulnar nerve decompression is as
effective as subcutaneous transposition for most symptomatic ulnar neuropathy. Ulnar
nerve instability before or after decompression may best be treated by transposition
rather than simple decompression. Submuscular transposition with or without medial
epicondylectomy may be best reserved for revision surgery or patients who are
exceedingly thin. Arthroscopic nerve decompression has been reported with
arthroscopic treatment of elbow arthritis, but is associated with a higher complication
and revision rate than the standard techniques. Adequate nerve decompression, rather
than duration of symptoms, is the most important predictor of outcome.
RECOMMENDED READINGS
1- Caliandro P, La Torre G, Padua R, Giannini F, Padua L. Treatment for ulnar neuropathy at the elbow. Cochrane
Database Syst Rev. 2012 Jul 11;7:CD006839. doi: 10.1002/14651858.CD006839.pub3. Review. PubMed MID:
22786500. View Abstract at PubMed
2- Zlowodzki M, Chan S, Bhandari M, Kalliainen L, Schubert W. Anterior transposition compared with simple
decompression for treatment of cubital tunnel syndrome. A meta-analysis of randomized, controlled trials. J
Bone Joint Surg Am. 2007 Dec;89(12):2591-8. PubMed PMID: 18056489.View Abstract at PubMed
3- Macadam SA, Gandhi R, Bezuhly M, Lefaivre KA. Simple decompression versus anterior subcutaneous and
submuscular transposition of the ulnar nerve for cubital tunnel syndrome: a meta-analysis. J Hand Surg Am.
2008 Oct;33(8):1314.e1-12. doi: 10.1016/j.jhsa.2008.03.006. Review. PubMed PMID: 18929194.View Abstract
at PubMed
4- Gellman H. Compression of the ulnar nerve at the elbow: cubital tunnel syndrome. Instr Course Lect.
2008;57:187-97. Review. PubMed PMID: 18399580.View Abstract at PubMed
5- Kovachevich R, Steinmann SP. Arthroscopic ulnar nerve decompression in the setting of elbow osteoarthritis. J
Hand Surg Am. 2012 Apr;37(4):663-8. doi: 10.1016/j.jhsa.2012.01.003. Epub 2012 Mar 3. PubMed PMID:
22386545.View Abstract at PubMed
6- Shi Q, MacDermid JC, Santaguida PL, Kyu HH. Predictors of surgical outcomes following anterior
transposition of ulnar nerve for cubital tunnel syndrome: a systematic review. J Hand Surg Am. 2011
Dec;36(12):1996-2001.e1-6. doi: 10.1016/j.jhsa.2011.09.024. Review. PubMed PMID: 22123047. View
Abstract at PubMed
Shoulder and Elbow Self-Assessment Examination AAOS 2014

Question 51 of 105
A 21-year-old college swimmer presents with an inability to compete for longer than 1
year because of right shoulder pain and subjective symptoms of instability despite
physical therapy. Recent radiographs and an MRI scan of her shoulder demonstrate an
intact labral complex. Her symptoms are reproduced with sulcus testing and load and
shift maneuvers in both anterior and posterior directions. What is the most appropriate
next treatment step?
1- Continued physical therapy
2- Open capsular shift
3- Arthroscopic capsulolabral shift
4- Thermal capsulorrhaphy

PREFERRED RESPONSE: 2- Open capsular shift


DISCUSSION
In this scenario, the patient has already failed therapy. An MRI scan did not indicate
recurrent injury to the labrum. The open capsular shift procedure initially popularized
by Neer and modified by Wirth and Rockwood allows surgeons to reduce joint volume
by imbricating the patulous inferior capsule through an anterior axillary-based
incision. Through this anterior approach, concomitant injuries such as a humeral
avulsion of the glenohumeral ligament, Bankart lesions, and subscapularis tears may
be addressed. Multidirectional instability (MDI) of the shoulder is defined as
symptomatic instability in more than 1 direction. Both dynamic (rotator cuff,
periscapular muscles) and static stabilizers (glenoid, labrum, and joint capsule) impart
stability to the glenohumeral joint. In this patient, the examination may indicate a
positive sulcus sign, increased humeral head translation in more than 1 direction with
the load-and-shift test, and generalized hyperlaxity such as elbow, knee, and
metacarpophalangeal joint hyperextension. Radiographs may infrequently reveal
glenoid dysplasia and bone loss. MRI arthrogram imaging may demonstrate a
redundant capsule without specific injury to the labral complex. Initial management of
MDI involves therapy with a focus on rotator cuff and periscapular muscle
strengthening. Arthroscopic capsulorrhaphy may be a viable option but is not
indicated in this scenario based upon lack of injury to the labrum. As with the open
capsular shift, care must be taken when performing an arthroscopic plication to avoid
overreduction and subsequent shoulder stiffness. Thermal caspulorrhaphy has been
abandoned as a treatment option because of high failure rate (60%) and risk for
chondrolysis and axillary nerve injury.
RECOMMENDED READINGS
Shoulder and Elbow Self-Assessment Examination AAOS 2014

1- Schenk TJ, Brems JJ. Multidirectional instability of the shoulder: pathophysiology, diagnosis, and management.
J Am Acad Orthop Surg. 1998 Jan-Feb;6(1):65-72. Review. PubMed PMID: 9692942.View Abstract at PubMed
2- Gaskill TR, Taylor DC, Millett PJ. Management of multidirectional instability of the shoulder. J Am Acad
Orthop Surg. 2011 Dec;19(12):758-67. Review. PubMed PMID: 22134208.View Abstract at PubMed
3- Bois AJ, Wirth MA. Revision open capsular shift for atraumatic and multidirectional instability of the shoulder.
J Bone Joint Surg Am. 2012 Apr 18;94(8):748-56. PubMed PMID: 22517392.View Abstract at PubMed

Question 52 of 105
A 15-year-old girl has experienced 6 months of increasing dominant shoulder pain
while playing volleyball. Her pain is so significant that she can no longer compete.
Examination demonstrates 190 degrees of forward elevation, 110 degrees of external
rotation at the side, and internal rotation up the back to T2 bilaterally. She also has 15
degrees of bilateral elbow hyperextension. Load and shift testing demonstrates pain
with anterior and posterior drawer tests. She has a large sulcus on examination that
causes pain during testing. Forward elevation and external rotation strength testing
shows 4/5 strength. There is no scapular winging and radiograph findings are normal.
What is the best next step?
1- Physical therapy for rotator cuff strengthening
2- Subacromial corticosteroid injection
3- MRI arthrogram
4- Arthroscopic stabilization

PREFERRED RESPONSE: 1- Physical therapy for rotator cuff strengthening


DISCUSSION
This patient has multidirectional instability as evidenced by her hyperlaxity and
excessive range of motion. Patients with pain after activities often have weak rotator
cuff musculature and improve with strengthening of the rotator cuff and
proprioceptive retraining. Subacromial injection likely cannot help this patient because
it will not treat her underlying motor weakness in the rotator cuff or her dyskinesia.
MRI arthrogram is not indicated unless she fails nonsurgical treatment. Arthroscopic
stabilization also would be reserved for patients who fail nonsurgical treatment.

RECOMMENDED READINGS
1- Gaskill TR, Taylor DC, Millett PJ. Management of multidirectional instability of the shoulder. J Am Acad
Orthop Surg. 2011 Dec;19(12):758-67. Review. PMID: 22134208.View Abstract at PubMed
2- Jacobson ME, Riggenbach M, Wooldridge AN, Bishop JY. Open capsular shift and arthroscopic capsular
plication for treatment of multidirectional instability. Arthroscopy. 2012 Jul;28(7):1010-7. Review. PMID:
22365265.View Abstract at PubMed
Shoulder and Elbow Self-Assessment Examination AAOS 2014

CLINICAL SITUATION FOR QUESTIONS 53 AND 54


Figures 53a and 53b are the radiographs of a 47-year-old right-hand-dominant active
man with a 10-year history of progressive right elbow pain associated with stiffness.
He previously underwent collateral ligament reconstruction. He has pain throughout
his range of motion arc, which currently measures 20 degrees of extension to 80
degrees of flexion. Initial treatment with nonsteroidal anti-inflammatory medication,
physical therapy, cortisone injections, and arthroscopic debridement has failed to
provide relief of his symptoms and improvement in function.

Question 53 of 105
What is the most appropriate next treatment step for this patient?
1- Total elbow arthroplasty (TEA)
2- Distal humeral replacement arthroplasty
3- Arthroscopic release with debridement
4- Soft-tissue interposition arthroplasty

PREFERRED RESPONSE: 4- Soft-tissue interposition arthroplasty

Question 54 of 105
What is the most appropriate treatment if instability is present at the time of
evaluation?
1- TEA
2- Distal humeral replacement arthroplasty
3- Arthroscopic release with debridement
4- Soft-tissue interposition arthroplasty

PREFERRED RESPONSE: 1- TEA


DISCUSSION
Shoulder and Elbow Self-Assessment Examination AAOS 2014

The radiographs reveal ulnohumeral arthrosis with relative sparing of the


radiocapitellar articulation secondary to underlying osteoarthritis. Arthrosis of the
elbow joint in this young and active patient presents a treatment dilemma for the
surgeon. Interposition arthroplasty allows for improved function with pain relief and
no weight-lifting restrictions, as required with TEA. This option is an intermediate
procedure that preserves bone stock and allows for conversion to a TEA if necessary.
Conventional TEA would provide pain relief with improved range of motion, but
activity limitation and lifetime weight restrictions make this an undesirable option.
Arthroscopic debridement is not an option, considering the previous failure from this
modality. Contraindications for soft-tissue interposition arthroplasty include elbow
instability, active infection, and pain without motion loss. Common complications
associated with this procedure include instability, infection, ulnar neuropathy, bone
resorption, and heterotopic bone formation.

RECOMMENDED READINGS
1- Larson AN, Morrey BF. Interposition arthroplasty with an Achilles tendon allograft as a salvage procedure for
the elbow. J Bone Joint Surg Am. 2008 Dec;90(12):2714-23. PubMed PMID: 19047718.View Abstract at
PubMed
2- Blaine TA, Adams R, Morrey BF. Total elbow arthroplasty after interposition arthroplasty for elbow arthritis. J
Bone Joint Surg Am. 2005 Feb;87(2):286-92. PubMed PMID: 15687149.View Abstract at PubMed
3- Celli A, Morrey BF. Total elbow arthroplasty in patients forty years of age or less. J Bone Joint Surg Am. 2009
Jun;91(6):1414-8. PubMed PMID: 19487519.View Abstract at PubMed
4- Surgical management of traumatic conditions of the elbow. In: Weisel S, Ramsey ML, eds. Operative
Techniques in Orthopaedic Surgery. Philadelphia, PA: Lippincott, Williams & Wilkins; 2011:3453-3461.

Question 55 of 105
Figures 55a and 55b are the radiographs of a 64-year-old woman with a history
significant for rheumatoid arthritis who has the chief complaint of right elbow pain.
She has been treated with tumor necrosis factor-alpha inhibitors and oral
corticosteroids for several years. The patient experiences severe global elbow pain and
crepitus. What process primarily is responsible for joint
destruction in rheumatoid arthritis?
Shoulder and Elbow Self-Assessment Examination AAOS 2014

1- Traumatic insult resulting in complement activation


2- Mutation in the rheumatoid factor gene
3- Osteoblast paracrine signaling resulting in proteolytic collagen degradation
4- Inflammation resulting in a hyperplastic synovial joint lining

PREFERRED RESPONSE: 4- Inflammation resulting in a hyperplastic synovial


joint lining
DISCUSSION
Rheumatoid arthritis is a systemic inflammatory disorder marked by erosive arthritis
in multiple joints. Elbow involvement is common. The pathologic lesion in
rheumatoid arthritis is pannus, a hyperplastic synovial proliferation that ultimately
results in proteoglycan and collagen digestion. Rheumatoid factor mutations, traumatic
insults resulting in complement activation and osteoblast paracrine signaling, are not
involved in the pathologic process. The Larsen classification assesses the progression
of rheumatoid changes in the elbow. Stage I is characterized by osteopenia without
joint space narrowing. Stage II indicates joint space narrowing, but a normal joint
contour. Stage III is marked by joint space loss. This patient has stage IV disease, as
seen by the advanced erosive changes with trochlear groove deepening and resulting
deformity. Stage V is ankylosis.

RECOMMENDED READINGS
1- Chen AL, Joseph TN, Zuckerman JD. Rheumatoid arthritis of the shoulder. J Am Acad Orthop Surg. 2003 Jan-
Feb;11(1):12-24. Review. PubMed PMID: 12699368.View Abstract at PubMed
2- Larsen A, Dale K, Eek M. Radiographic evaluation of rheumatoid arthritis and related conditions by standard
reference films. Acta Radiol Diagn (Stockh). 1977 Jul;18(4):481-91. PubMed PMID: 920239.View Abstract at
PubMed
3- Gill DR, Morrey BF. The Coonrad-Morrey total elbow arthroplasty in patients who have rheumatoid arthritis. A
ten to fifteen-year follow-up study. J Bone Joint Surg Am. 1998 Sep;80(9):1327-35. PubMed PMID: 9759818.
View Abstract at PubMed

CLINICAL SITUATION FOR QUESTIONS 56 THROUGH 58


Figure 56 is the radiograph of a 47-year-old woman who has pain and difficulty
raising her arm after playing 36 holes of golf in a weekend. She denies prior episodes
of shoulder pain, and now has difficulty sleeping. Examination demonstrates guarding
with any shoulder motion, tenderness around the superolateral shoulder, and normal
sensory findings.
Shoulder and Elbow Self-Assessment Examination AAOS 2014

Question 56 of 105
An MRI arthrogram scan of her shoulder would show
1- increased T2 signal in the rotator cuff.
2- fluid escape into the subacromial space.
3- fluid in the glenoid/labral fissure.
4- isointense signal to the rotator cuff.

PREFERRED RESPONSE: 4- isointense signal to the rotator cuff.

Question 57 of 105
The lesion indicated in the image is made of
1- calcium carbonate apatite.
2- hyperproliferative white blood cells.
3- hydroxyapatite crystals.
4- degenerated tenocytes.

PREFERRED RESPONSE: 1- calcium carbonate apatite.

Question 58 of 105
The best initial treatment would entail
1- physical therapy and nonsteroidal anti-inflammatory medications.
2- open biopsy of the lesion for permanent section.
3- manipulation under anesthesia.
4- shoulder arthroscopy.
Shoulder and Elbow Self-Assessment Examination AAOS 2014

PREFERRED RESPONSE: 1- physical therapy and nonsteroidal anti-inflammatory


medications.
DISCUSSION
Calcific tendinitis of the shoulder is a deposition of calcium carbonate apatite crystals
into the structure of the rotator cuff tendon. The crystalline form appears to progress
throughout the clinical disease process, demonstrating increasing matured
stoichiometric apatite deposition during the resorptive phase. MRI can be difficult to
interpret because the signal of the calcific lesion is frequently similar to that seen in
normal supraspinatus tendon. Plain radiographs remain the gold standard for
diagnosis. Ultrasound can be a useful ancillary study to determine the location and
size of the lesion. Primary management of calcific tendinitis starts with nonsurgical
treatment including physiotherapy and injections, if indicated. Mixed results have
been reported with extracorporeal shock wave therapy. Surgical removal with repair of
the tendon in larger lesions remains the definitive treatment when nonsurgical
modalities fail. Subacromial decompression may improve pain relief in patients who
require surgery; however, patients with decompression may take longer to fully
recover.
RECOMMENDED READINGS
1- Zubler C, Mengiardi B, Schmid MR, Hodler J, Jost B, Pfirrmann CW. MR arthrography in calcific tendinitis of
the shoulder: diagnostic performance and pitfalls. Eur Radiol. 2007 Jun;17(6):1603-10. Epub 2006 Oct 12.
PMID: 17036154.View Abstract at PubMed
2- Oliva F, Via AG, Maffulli N. Physiopathology of intratendinous calcific deposition. BMC Med. 2012 Aug
23;10:95. doi: 10.1186/1741-7015-10-95. PubMed PMID: 22917025.View Abstract at PubMed
3- Ark JW, Flock TJ, Flatow EL, Bigliani LU. Arthroscopic treatment of calcific tendinitis of the shoulder.
Arthroscopy. 1992;8(2):183-8. PubMed PMID: 1637430.View Abstract at PubMed
4- Marder RA, Heiden EA, Kim S. Calcific tendonitis of the shoulder: is subacromial decompression in
combination with removal of the calcific deposit beneficial? J Shoulder Elbow Surg. 2011 Sep;20(6):955-60.
Epub 2011 Feb 1. PMID: 21277805.View Abstract at PubMed
5- Balke M, Bielefeld R, Schmidt C, Dedy N, Liem D. Calcifying tendinitis of the shoulder: midterm results after
arthroscopic treatment. Am J Sports Med. 2012 Mar;40(3):657-61. Epub 2011 Dec 8. PMID: 22156173.View
Abstract at PubMed

Question 59 of 105
Figure 59 is the MRI scan of a 30-year-old fire fighter who dislocated his left shoulder
during work activities. His shoulder was reduced in the emergency department. After
8 weeks of physical therapy, he continues to have apprehension when lifting and
pushing the fire hose back into the truck. He has normal rotator cuff strength and a
negative sulcus sign. What treatment option will allow this patient to return to work as
soon as possible?
Shoulder and Elbow Self-Assessment Examination AAOS 2014

1- Strengthening in physical therapy


2- Anterior labral repair
3- Posterior labral repair
4- Coracoid transfer

PREFERRED RESPONSE: 3- Posterior labral repair


DISCUSSION
The MRI scan shows a posterior labral tear, which is contributing to his posterior
instability. If a patient does not improve after 8 weeks of physical therapy, the therapy
likely will not correct his or her instability. Activities that involve internal rotation of
the shoulder and adduction will put the shoulder at most risk for posterior dislocation
or subluxation. The patient elevates his risk when he pushes fire hoses into the truck
repetitively, which further supports the diagnosis of posterior instability attributable to
a posterior labral tear. An anterior labral repair or a coracoid transfer would not treat
posterior instability.
RECOMMENDED READINGS
1- Lenart BA, Sherman SL, Mall NA, Gochanour E, Twigg SL, Nicholson GP. Arthroscopic repair for posterior
shoulder instability. Arthroscopy. 2012 Oct;28(10):1337-43. doi: 10.1016/j.arthro.2012.03.011. Epub 2012 Jun
15. PubMed PMID: 22705299.View Abstract at PubMed
2- Provencher MT, LeClere LE, King S, McDonald LS, Frank RM, Mologne TS, Ghodadra NS, Romeo AA.
Posterior instability of the shoulder: diagnosis and management. Am J Sports Med. 2011 Apr;39(4):874-86. doi:
10.1177/0363546510384232. Epub 2010 Dec 4. PubMed PMID: 21131678.View Abstract at PubMed

CLINICAL SITUATION FOR QUESTIONS 60 AND 61


A 10-year-old left-hand-dominant baseball pitcher has had left elbow pain for 6
weeks. His pain primarily is located medially, and he states that it is worst during the
late cocking/early acceleration phase of his pitch. Recently he noticed that he is not
able to throw as fast as usual. He decreased his pitch count by half during the last 2
weeks without significant improvement in his symptoms. When he is not pitching, he
does not have significant pain. Radiographs show widening of the medial epicondyle
physis.
Shoulder and Elbow Self-Assessment Examination AAOS 2014

Question 60 of 105
At what age does the medial epicondyle epiphysis ossification center appear and then
fuse?
1- Appears at 2 to 3 years, fuses at 12 to 13 years
2- Appears at 5 to 6 years, fuses at 15 to 16 years
3- Appears at 6 to 8 years, fuses at 12 to 13 years
4- Appears at 8 to 10 years, fuses at 15 to 16 years

PREFERRED RESPONSE: 2- Appears at 5 to 6 years, fuses at 15 to 16 years

Question 61 of 105
What biomechanical forces and pathology most likely underlie this patient’s pain and
injury?
1- Acute avulsion of the medial epicondyle attributable to valgus stress
2- Chronic weakening of the ulnar collateral ligament attributable to chronic
tension forces
3- Chronic compressive forces on the medial epicondyle leading to
fragmentation
4- Chronic tension forces of valgus overload on the medial epicondyle leading
to physeal separation

PREFERRED RESPONSE: 4- Chronic tension forces of valgus overload on the


medial epicondyle leading to physeal separation
DISCUSSION
The medial epicondyle ossification center is first seen at 5 to 6 years of age and is the
last to fuse at age 15 to 16. The capitellum first appears at age 1 to 2. The radial
epicondylar epiphysis appears at 2 to 4 years. The trochlea appears at age 8 to 10; the
olecranon appears at approximately age 10. The lateral epicondylar epiphysis is the
last to appear at age 12.
Repetitive tensile stress on the medial epicondyle is caused by the flexor-pronator
mass and the ulnar collateral ligament. This chronic valgus microtrauma eventually
can lead to apophysitis and/or stress fracture. In the skeletally immature athlete,
tension across the medial elbow produces a physeal injury rather than a ligamentous
injury. Based upon the history, this is a repetitive overuse injury and not an acute
injury.

RECOMMENDED READINGS
Shoulder and Elbow Self-Assessment Examination AAOS 2014

1- Klingele KE, Kocher MS. Little league elbow: valgus overload injury in the pediatric athlete. Sports Med.
2002;32(15):1005-15. Review. PubMed PMID: 12457420.View Abstract at PubMed
2- Rudzki JR, Paletta GA Jr. Juvenile and adolescent elbow injuries in sports. Clin Sports Med. 2004
Oct;23(4):581-608, ix. Review. PubMed PMID: 15474224.View Abstract at PubMed

Question 62 of 105
A 35-year-old man fell off of a roof and sustained an extra-articular supracondylar
elbow fracture. He had normal sensation in all fingers after the injury and before
undergoing surgery to repair the fracture. The ulnar nerve was not transposed, but it
was inspected prior to wound closure. Ten days after surgery, the patient has
numbness in his small finger and is unable to cross his fingers. His elbow range of
motion is between 40 degrees and 100 degrees. What is the next appropriate treatment
step?
1- Elbow splint at 40 degrees at night for 6 weeks
2- Electromyography (EMG)
3- Exploration of the ulnar nerve and transposition
4- Observation

PREFERRED RESPONSE: 4- Observation


DISCUSSION
This patient has an early postsurgical ulnar nerve palsy. The causes of this injury are
laceration of the nerve during surgery, entrapment of the nerve in the fracture or
hardware, or traction injury during surgery. If the orthopaedic surgeon is sure that the
nerve was not lacerated at the end of the case or entrapped in the hardware, then the
nerve is probably intact and will recover. Observation is the best treatment in this case
because the nerve was checked before wound closure. Elbow splinting has not been
shown to help with postsurgical nerve recovery. EMG findings may not be accurate
this early in the injury.

RECOMMENDED READINGS
1- Shin R, Ring D. The ulnar nerve in elbow trauma. J Bone Joint Surg Am. 2007 May;89(5):1108-16. Review.
PubMed PMID: 17473151.View Abstract at PubMed
2- Faierman E, Wang J, Jupiter JB. Secondary ulnar nerve palsy in adults after elbow trauma: a report of two cases.
J Hand Surg Am. 2001 Jul;26(4):675-8. PubMed PMID: 11466643.View Abstract at PubMed

Question 63 of 105
A 54-year-old pipefitter falls from a ladder at work and dislocates his nondominant
shoulder. His MRI scan shows supraspinatus and infraspinatus tears with retraction to
Shoulder and Elbow Self-Assessment Examination AAOS 2014

the glenoid. He cannot actively raise his arm away from his side. He denies prior
shoulder symptoms before his fall. Three weeks of physical therapy have failed to
improve his function. You and the patient decide to proceed with surgical repair.
Which is a risk factor for a poor outcome?
1- The patient’s age
2- The patient’s gender
3- Work-related injury
4- Acute nature of the tear

PREFERRED RESPONSE: 3- Work-related injury


DISCUSSION
Several studies have demonstrated that patients with work-related injuries do not do as
well as those whose injuries are not work-related after repair of the rotator cuff. This
patient’s age and gender are not negative prognostic indicators. The acute nature of the
tear does not lead to an inferior outcome.

RECOMMENDED READINGS
1- Kemp KA, Sheps DM, Luciak-Corea C, Styles-Tripp F, Buckingham J, Beaupre LA. Systematic review of
rotator cuff tears in workers' compensation patients. Occup Med (Lond). 2011 Dec;61(8):556-62. Epub 2011
Oct 19. Review. PMID: 22016341.View Abstract at PubMed
2- Bhatia S, Piasecki DP, Nho SJ, Romeo AA, Cole BJ, Nicholson GP, Boniquit N, Verma NN. Early return to
work in workers' compensation patients after arthroscopic full-thickness rotator cuff repair. Arthroscopy. 2010
Aug;26(8):1027-34. Epub 2010 Jun 3. PMID: 20678699.View Abstract at PubMed

RESPONSES FOR QUESTIONS 64 THROUGH 68


1- Internal rotation stretching, core stability exercises, and scapular stabilization
exercises
2- Arthroscopic debridement
3- Arthroscopic debridement with subacromial decompression
4- Arthroscopic transtendinous repair
5- Arthroscopic tear completion and repair

Please match the interventions above to the scenarios below.

Question 64 of 105
A 23-year-old Division 1 baseball pitcher is experiencing worsening pain despite
completion of an extensive, but unsuccessful, sport-specific physical therapy regimen.
Shoulder and Elbow Self-Assessment Examination AAOS 2014

An MRI scan shows articular surface tearing of the rotator cuff and internal
impingement on abduction external rotation views.
1- Internal rotation stretching, core stability exercises, and scapular stabilization
exercises
2- Arthroscopic debridement
3- Arthroscopic debridement with subacromial decompression
4- Arthroscopic transtendinous repair
5- Arthroscopic tear completion and repair

PREFERRED RESPONSE: 2- Arthroscopic debridement

Question 65 of 105
A 55-year-old woman with a bursal-sided tear less than 20% thickness and lateral
acromial impingement has failed physical therapy.
1- Internal rotation stretching, core stability exercises, and scapular stabilization
exercises
2- Arthroscopic debridement
3- Arthroscopic debridement with subacromial decompression
4- Arthroscopic transtendinous repair
5- Arthroscopic tear completion and repair

PREFERRED RESPONSE: 3- Arthroscopic debridement with subacromial


decompression

Question 66 of 105
A 55-year-old man with worsening night pain has no history of trauma, and
examination demonstrates posterior capsular tightness and scapular dyskinesia.
Radiograph findings appear normal.
1- Internal rotation stretching, core stability exercises, and scapular stabilization
exercises
2- Arthroscopic debridement
3- Arthroscopic debridement with subacromial decompression
4- Arthroscopic transtendinous repair
5- Arthroscopic tear completion and repair

PREFERRED RESPONSE: 1- Internal rotation stretching, core stability exercises,


and scapular stabilization exercises
Shoulder and Elbow Self-Assessment Examination AAOS 2014

Question 67 of 105
A 17-year-old high school pitcher has increasing pain accompanied by decreased ball
velocity.
1- Internal rotation stretching, core stability exercises, and scapular stabilization
exercises
2- Arthroscopic debridement
3- Arthroscopic debridement with subacromial decompression
4- Arthroscopic transtendinous repair
5- Arthroscopic tear completion and repair

PREFERRED RESPONSE: 1- Internal rotation stretching, core stability exercises,


and scapular stabilization exercises

Question 68 of 105
A 65-year-old man who has failed nonsurgical treatment demonstrates a partial-
thickness supraspinatus tendon tear of 70% thickness.
1- Internal rotation stretching, core stability exercises, and scapular stabilization
exercises
2- Arthroscopic debridement
3- Arthroscopic debridement with subacromial decompression
4- Arthroscopic transtendinous repair
5- Arthroscopic tear completion and repair

PREFERRED RESPONSE: 5- Arthroscopic tear completion and repair


DISCUSSION
For the young athlete, a careful examination of scapular mechanics, core stability, and
internal rotation deficits is important to diagnose, prevent, and treat a painful shoulder.
Correction of the capsular contracture and core imbalance often is enough to alleviate
symptoms. If nonsurgical management fails, MRI scan findings and arthroscopic
examination often show partial-thickness tearing of the articular surface of the
supraspinatus or infraspinatus with or without associated internal impingement or
aberrant contact with the posterior superior labrum and glenoid. The prevalence may
be as high as 20% to 40% in the overhead athletic population, likely resulting from
repetitive microtrauma. Degenerative tears often become symptomatic with an
increase in size; the mechanical linkage between the supraspinatus and infraspinatus
makes it likely that the remaining fibers bear more strain as tear size increases. Simple
acromioplasty may be successful only in cases of definitive contact between the
acromion and supraspinatus. Transtendinous repairs are effective treatment for partial-
Shoulder and Elbow Self-Assessment Examination AAOS 2014

thickness tears of the supraspinatus, but they may have a slower functional recovery
and a higher rate of stiffness than excision and repair; this may be attributable, in part,
to the natural overlap of the infraspinatus tendon over the supraspinatus tendon. An
“all-inside” technique may be preferable in younger patients because it is possible to
reinsert only the surface fibers that are torn, avoiding constraint of the superficial,
bursal fibers. For tears of more than 50% thickness, completing the tear to excise the
remaining degenerative fibers may be the preferred treatment.

RECOMMENDED READINGS
1- Ide J, Maeda S, Takagi K. Arthroscopic transtendon repair of partial-thickness articular-side tears of the rotator
cuff: anatomical and clinical study. Am J Sports Med. 2005 Nov;33(11):1672-9. Epub 2005 Aug 10. PubMed
PMID: 16093533.View Abstract at PubMed
2- Yang S, Park HS, Flores S, Levin SD, Makhsous M, Lin F, Koh J, Nuber G, Zhang LQ. Biomechanical analysis
of bursal-sided partial thickness rotator cuff tears. J Shoulder Elbow Surg. 2009 May-Jun;18(3):379-85. doi:
10.1016/j.jse.2008.12.011. Epub 2009 Mar 9. PubMed PMID: 19269860.View Abstract at PubMed
3- Finnan RP, Crosby LA. Partial-thickness rotator cuff tears. J Shoulder Elbow Surg. 2010 Jun;19(4):609-16. doi:
10.1016/j.jse.2009.10.017. Epub 2010 Feb 19. Review. PubMed PMID: 20171904.View Abstract at PubMed
4- Kamath G, Galatz LM, Keener JD, Teefey S, Middleton W, Yamaguchi K. Tendon integrity and functional
outcome after arthroscopic repair of high-grade partial-thickness supraspinatus tears. J Bone Joint Surg Am.
2009 May;91(5):1055-62. doi: 10.2106/JBJS.G.00118. Erratum in: J Bone Joint Surg Am. 2009
Aug;91(8):1995. PubMed PMID: 19411453.View Abstract at PubMed

Question 69 of 105
A 17-year-old left-hand-dominant gymnast has a 10-week history of gradually
progressive right shoulder pain. She reports the onset of pain to be associated with an
increase in her training regimen while preparing for an upcoming regional
competition, and denies any specific trauma to her shoulder. Examination reveals end-
range discomfort, but normal active and passive range of motion. Her periscapular
musculature strength is normal, but she demonstrates mild medial scapular winging
with arm elevation. She has 20 degrees’ elbow recurvatum, a positive sulcus
examination, and can hyperextend the metacarpophalangeal joint of her index finger to
105 degrees. What is the most appropriate initial treatment?
1- Physical therapy referral for rotator cuff and periscapular conditioning
2- Electromyography
3- Subacromial injection
4- Arthroscopic capsular plication

PREFERRED RESPONSE: 1- Physical therapy referral for rotator cuff and


periscapular conditioning
DISCUSSION
Shoulder and Elbow Self-Assessment Examination AAOS 2014

This patient has shoulder pain and an underlying diagnosis of multidirectional


glenohumeral laxity. There remains controversy as to whether athletes with features of
generalized laxity are at increased risk for shoulder pain. This athlete’s presentation is
typical in that the onset was atraumatic and associated with an increase in her training
regimen. The most appropriate treatment step is rest from competition and institution
of a dedicated physical therapeutic exercise program that emphasizes rotator cuff and
periscapular strengthening, with a focus on the serratus anterior. Electromyography
would not be helpful because the patient’s isolated periscapular motor function is
intact and she demonstrates a typical pattern of acquired scapular dyskinesis seen in
painful overhead athletic shoulder. Subacromial injection therapy is not indicated in
the setting of multidirectional instability. Arthroscopic surgical options are considered
as a final treatment intervention after nonsurgical measures have failed, and
appropriate imaging, such as MRI scan, has been obtained to determine the presence
or absence of significant structural abnormalities.
RECOMMENDED READINGS
1- Bak K. The practical management of swimmer's painful shoulder: etiology, diagnosis, and treatment. Clin J
Sport Med. 2010 Sep;20(5):386-90. doi: 10.1097/JSM.0b013e3181f205fa. PubMed PMID: 20818199. View
Abstract at PubMed
2- Madsen PH, Bak K, Jensen S, Welter U. Training induces scapular dyskinesis in pain-free competitive
swimmers: a reliability and observational study. Clin J Sport Med. 2011 Mar;21(2):109-13. doi: 10.1097/JSM.
0b013e3182041de0. PubMed PMID: 21358500.View Abstract at PubMed

CLINICAL SITUATION FOR QUESTIONS 70 THROUGH 72


A 17-year-old high school football player sustains a traumatic anterior shoulder
dislocation resulting in a small bony Bankart lesion and small Hill-Sachs lesion. The
patient undergoes an arthroscopic Bankart repair with incorporation of the bone
fragment and returns to play football the following year. He has a recurrent dislocation
at football practice, but decides to finish the football season before considering
additional treatment. He sustains 9 additional dislocations, with the last dislocation
occurring while sleeping.

Question 70 of 105
What diagnostic test is best when planning revision surgery?
1- CT scan with 3-dimensional (3-D) reconstructions
2- Ultrasound
3- MRI scan
4- Arthrogram

PREFERRED RESPONSE: 1- CT scan with 3-dimensional (3-D) reconstructions


Shoulder and Elbow Self-Assessment Examination AAOS 2014

Question 71 of 105
The patient has eroded one-third of the inferior glenoid surface area. What is the most
appropriate treatment?
1- Revision arthroscopic Bankart repair with capsular shift
2- Open Bankart repair with capsular shift
3- Repair of infraspinatus tendon into the Hill-Sachs defect (remplissage
procedure)
4- Coracoid transfer to the glenoid (Latarjet procedure)

PREFERRED RESPONSE: 4- Coracoid transfer to the glenoid (Latarjet procedure)

Question 72 of 105
Which patients are clinically most dissatisfied after revision instability surgery?
1- Patients with pain before surgery
2- Patients younger than 25 years of age
3- Patients older than 55 years of age
4- Recreational athletes

PREFERRED RESPONSE: 1- Patients with pain before surgery


DISCUSSION
A failed bony Bankart repair with multiple dislocations can further erode the
anteroinferior glenoid, changing the sagittal morphology of the glenoid into an
“inverted pear.” Quantitative bone loss is best evaluated by CT scan with 3-D
reconstructions and subtraction of the humeral head. MRI and ultrasound can assist in
evaluating soft-tissue injury, but they are not as helpful in determining bone loss
compared to a CT scan. An arthrogram alone is not sufficient to evaluate bone loss.
Bone loss exceeding 30% necessitates glenoid augmentation with either a Latarjet
procedure or iliac crest bone grafting. A revision arthroscopic or open Bankart repair
with capsular shift or remplissage do not address bone loss. The Latarjet procedure
can effectively restore stability with glenoid bone loss and after failed stabilizing
procedures. Patients with pain before surgery are more likely to have pain after
surgery. Age and activity level are lesser influences on satisfaction.
RECOMMENDED READINGS
1- Schmid SL, Farshad M, Catanzaro S, Gerber C. The Latarjet procedure for the treatment of recurrence of
anterior instability of the shoulder after operative repair: a retrospective case series of forty-nine consecutive
patients. J Bone Joint Surg Am. 2012 Jun 6;94(11):e75. doi: 10.2106/JBJS.K.00380. PubMed PMID: 22637215.
View Abstract at PubMed
2- Piasecki DP, Verma NN, Romeo AA, Levine WN, Bach BR Jr, Provencher MT. Glenoid bone deficiency in
recurrent anterior shoulder instability: diagnosis and management. J Am Acad Orthop Surg. 2009
Aug;17(8):482-93. Review. PubMed PMID: 19652030.View Abstract at PubMed
Shoulder and Elbow Self-Assessment Examination AAOS 2014

3- Burkhart SS, De Beer JF. Traumatic glenohumeral bone defects and their relationship to failure of arthroscopic
Bankart repairs: significance of the inverted-pear glenoid and the humeral engaging Hill-Sachs lesion.
Arthroscopy. 2000 Oct;16(7):677-94. PubMed PMID: 11027751.View Abstract at PubMed

Question 73 of 105
Figure 73 is the radiograph of a 78-year-old man who has had 8 months of gradually
progressive right shoulder pain. He temporarily responds to a corticosteroid injection
administered by his primary physician, but his symptoms quickly return. He reports
significant interference with activities of daily living and recreational activities.
Examination demonstrates active range of motion to 90 degrees’ forward elevation, 20
degrees’ external rotation at the side, and 50 degrees’
in the abducted position, with pain at end range. The
most appropriate next treatment step is

1- Arthroscopic glenohumeral debridement,


synovectomy, and biceps tenotomy
2- Total shoulder arthroplasty (TSA)
3- Reverse TSA (rTSA)
4- Humeral head arthroplasty without glenoid
resurfacing

PREFERRED RESPONSE: 3- Reverse TSA (rTSA)


DISCUSSION
This patient presents with a clinical history, examination, and imaging consistent with
end-stage rotator cuff tear arthropathy. Recommended treatment is rTSA. With
significantly limited function and advanced radiographic changes, arthroscopic
intervention is unlikely to provide significant clinical benefit. TSA, with or without
rotator cuff repair, likely would lead to early mechanical failure of the glenoid
component (edge loading, or “rocking horse” glenoid). Hemiarthroplasty was
previously the recommended treatment option, prior to the reintroduction of the
reverse implant. However, current data suggest reverse arthroplasty provides a more
predictable outcome (pain relief, improved function) and less need for surgical
revision.
RECOMMENDED READINGS
1- Cuff D, Pupello D, Virani N, Levy J, Frankle M. Reverse shoulder arthroplasty for the treatment of rotator cuff
deficiency. J Bone Joint Surg Am. 2008 Jun;90(6):1244-51. doi: 10.2106/JBJS.G.00775. PubMed PMID:
18519317.View Abstract at PubMed
Shoulder and Elbow Self-Assessment Examination AAOS 2014

2- Drake GN, O'Connor DP, Edwards TB. Indications for reverse total shoulder arthroplasty in rotator cuff disease.
Clin Orthop Relat Res. 2010 Jun;468(6):1526-33. doi: 10.1007/s11999-009-1188-9. Review. PubMed PMID:
20049573.View Abstract at PubMed
3- Leung B, Horodyski M, Struk AM, Wright TW. Functional outcome of hemiarthroplasty compared with reverse
total shoulder arthroplasty in the treatment of rotator cuff tear arthropathy. J Shoulder Elbow Surg. 2012
Mar;21(3):319-23 . doi: 10.1016/j.jse.2011.05.023. Epub 2011 Aug 26. PubMed PMID: 21872496. View
Abstract at PubMed

CLINICAL SITUATION FOR QUESTIONS 74 AND 75


Figures 74a and 74b are the radiographs of a 20-year-old collegiate football player
who has had recurring episodes of right shoulder instability after undergoing an
arthroscopic capsulolabral repair 1 year ago. Clinically, he demonstrates a positive
anterior apprehension test.

Question 74 of 105
What is the most appropriate diagnostic test?
1- MRI scan
2- Ultrasound
3- CT scan
4- Electromyogram and nerve conduction study

PREFERRED RESPONSE: 3- CT scan

Question 75 of 105
If nonsurgical treatment has failed, what surgical procedure will best reduce the risk
for recurrent instability?
1- Diagnostic shoulder arthroscopy with labral repair
2- Diagnostic shoulder arthroscopy with open capsular shift
Shoulder and Elbow Self-Assessment Examination AAOS 2014

3- Diagnostic shoulder arthroscopy with coracoid transfer


4- Diagnostic shoulder arthroscopy with thermal capsulorrhaphy

PREFERRED RESPONSE: 3- Diagnostic shoulder arthroscopy with coracoid


transfer
DISCUSSION
If nonsurgical treatment has failed in the revision setting, the amount of bone loss
should be quantified. The current standard for quantification of glenoid bone loss is
CT scan with or without digital subtraction of the humeral head. An initial diagnostic
arthroscopy may permit calculation of glenoid bone loss. The glenoid bare-spot
method popularized by Lo and associates provides a reliable estimate of bone loss.
Percentage of bone loss is calculated by subtracting the distance from the anterior rim
to the bare spot from the posterior rim-bare spot distance divided by twice the
posterior rim-bare spot distance. The critical limits of glenoid bone loss are based on a
combination of cadaveric and clinical reports. Nonsurgical management may still be a
reasonable choice with less than 20% glenoid bone loss in low-demand individuals,
patients with high surgical risk secondary to medical comorbidities, and voluntary
dislocators. When addressing recurrent anterior instability of the shoulder, it is
imperative to assess both soft-tissue and bone injury. Particular attention must be paid
to glenoid and humeral head deficiencies. Patient-specific demands should be
considered when discussing treatment options. In a high-demand patient such as this
contact athlete, surgical treatment is appropriate. In general, if glenoid bone loss is less
than 15%, a soft-tissue stabilization procedure may be all that is necessary. In those
with 15% to 25% bone loss, arthroscopic stabilization with bone fragment
incorporation may be performed if local bone is available. In the setting of a high-
demand patient with no local bone for repair, coracoid transfer, iliac crest bone
autograft, or distal tibial allograft is appropriate. With more than 25% bone loss, the
glenoid deficiency must be addressed. In this scenario, the athlete demonstrates more
than 25% bone loss involving the anteroinferior glenoid. This deficiency must be
addressed to restore stability to the glenohumeral joint. In a high-demand patient
(contact athlete), augmentation with iliac crest bone graft, distal tibial allograft, or a
coracoid transfer procedure is appropriate if local bone is not available.
RECOMMENDED READINGS
1- Boileau P, Villalba M, Héry JY, Balg F, Ahrens P, Neyton L. Risk factors for recurrence of shoulder instability
after arthroscopic Bankart repair. J Bone Joint Surg Am. 2006 Aug;88(8):1755-63. PubMed PMID: 16882898.
View Abstract at PubMed
2- Provencher MT, Bhatia S, Ghodadra NS, Grumet RC, Bach BR Jr, Dewing CB, LeClere L, Romeo AA.
Recurrent shoulder instability: current concepts for evaluation and management of glenoid bone loss. J Bone
Joint Surg Am. 2010 Dec;92 Suppl 2:133-51. Review. PubMed PMID: 21123597.View Abstract at PubMed
Shoulder and Elbow Self-Assessment Examination AAOS 2014

3- Piasecki DP, Verma NN, Romeo AA, Levine WN, Bach BR Jr, Provencher MT. Glenoid bone deficiency in
recurrent anterior shoulder instability: diagnosis and management. J Am Acad Orthop Surg. 2009
Aug;17(8):482-93. Review. PubMed PMID: 19652030.View Abstract at PubMed
4- Hovelius L, Sandström B, Olofsson A, Svensson O, Rahme H. The effect of capsular repair, bone block healing,
and position on the results of the Bristow-Latarjet procedure (study III): long-term follow-up in 319 shoulders. J
Shoulder Elbow Surg. 2012 May;21(5):647-60. Epub 2011 Jun 29. PubMed PMID: 21719316.View Abstract at
PubMed

Question 76 of 105
A 33-year old man sustains a posterior elbow dislocation after a fall. Attempts at
closed reduction result in recurrent instability. What is the most common ligamentous
injury found at the time of surgical stabilization?
1-Midsubstance tear of the lateral ulnar collateral ligament
2- Proximal avulsion of the ulnar collateral ligament
3- Proximal avulsion of the lateral ulnar collateral ligament
4- Distal bony avulsion of the ulnar collateral ligament from the sublime
tubercle

PREFERRED RESPONSE: 3- Proximal avulsion of the lateral ulnar collateral


ligament
DISCUSSION
Classic posterior elbow dislocations result from a posterolateral rotatory mechanism,
whereby the hand is fixed (typically on the ground) while the weight of the body
creates a valgus and external rotation moment on the elbow. This results first in
tearing of the lateral collateral ligament that proceeds medially through the anterior
and posterior joint capsules, ending with potential involvement of the ulnar collateral
ligament (but this is not universal). McKee and associates assessed the lateral soft-
tissue injury pattern of elbow dislocations with and without associated fractures at the
time of surgery. Injury to the lateral collateral ligament complex was seen in every
case, with avulsion from the distal humerus as the most common finding.
Midsubstance tears, proximal avulsions, and distal bony avulsions of the ulnar
collateral ligament are less common.
RECOMMENDED READINGS
1- McKee MD, Schemitsch EH, Sala MJ, O'driscoll SW. The pathoanatomy of lateral ligamentous disruption in
complex elbow instability. J Shoulder Elbow Surg. 2003 Jul-Aug;12(4):391-6. PubMed PMID: 12934037.
View Abstract at PubMed
2- O'Driscoll SW, Morrey BF, Korinek S, An KN. Elbow subluxation and dislocation. A spectrum of instability.
Clin Orthop Relat Res. 1992 Jul;(280):186-97. PubMed PMID: 1611741.View Abstract at PubMed
Shoulder and Elbow Self-Assessment Examination AAOS 2014

Question 77 of 105
A 25-year-old man is planning to have an elbow contracture release. His elbow range
of motion is 40 degrees to 90 degrees of flexion. He has no heterotopic ossification.
His ring and small fingers become numb as his elbow approaches his flexion endpoint.
There is no evidence of instability of the ulna-humeral or radioulnar joints. To achieve
the best possible outcome, the surgeon should

1-include postsurgical elbow continuous passive motion (CPM).


2-perform the surgery open.
3- decompress the ulnar nerve.
4- release the anterior band of the medial collateral ligament.

PREFERRED RESPONSE: 3- decompress the ulnar nerve.


DISCUSSION
The patient is exhibiting signs of ulnar neuropathy. The surgeon should be sure to
decompress and possibly transpose the ulnar nerve, if unstable, to prevent worsening
neuropathy after surgery. CPM has not been shown to be of benefit after contracture
release. Equal success rates have been shown for open and arthroscopic contracture
releases. The anterior band of the medial collateral ligament is important to maintain
valgus stability of the elbow. The posterior band can be released to improve flexion
without increasing concern for elbow instability.

RECOMMENDED READINGS
1- Charalambous CP, Morrey BF. Posttraumatic elbow stiffness. J Bone Joint Surg Am. 2012 Aug 1;94(15):1428-
37. doi: 10.2106/JBJS.K.00711. Review. PubMed PMID: 22854997.View Abstract at PubMed
2- Williams BG, Sotereanos DG, Baratz ME, Jarrett CD, Venouziou AI, Miller MC. The contracted elbow: is ulnar
nerve release necessary? J Shoulder Elbow Surg. 2012 Jun 26. [Epub ahead of print] PubMed PMID: 22743068.
View Abstract at PubMed
3- Lindenhovius AL, van de Luijtgaarden K, Ring D, Jupiter J. Open elbow contracture release: postoperative
management with and without continuous passive motion. J Hand Surg Am. 2009 May-Jun;34(5):858-65. Epub
2009 Apr 11. PubMed PMID: 19362791.View Abstract at PubMed

Question 78 of 105
Figures 78a and 78b are the radiographs of a 47-year-old right-hand-dominant woman
who has a 3-month history of gradually progressive right shoulder pain. She reports no
previous trauma, but does report pain at night and with activity such as weight
training. Examination demonstrates active and passive range of motion to be 110
degrees forward elevation, external rotation to 20 degrees, and internal rotation to the
sacrum. The next treatment step should include
Shoulder and Elbow Self-Assessment Examination AAOS 2014

1- an MRI of the shoulder.


2- a physical therapy referral for rotator cuff strengthening and proprioceptive
exercise.
3- a home stretching program and corticosteroid injection.
4- arthroscopic glenohumeral capsular release.

PREFERRED RESPONSE: 3- a home stretching program and corticosteroid


injection.
DISCUSSION
This patient has idiopathic adhesive glenohumeral stiffness. Most patients with this
condition are women between 40 and 60 years of age with no specific mechanism of
onset. Patients typically develop pain, at which point the disease is marked by
significant inflammation. This patient is likely in the second stage of the disease,
marked by inflammation and early fibrosis of the joint capsule, leading to joint
stiffness. Associated conditions include diabetes mellitus and hypothyroidism,
although there is no explainable cause for most cases. The most appropriate treatment
step at this stage is an intra-articular glenohumeral corticosteroid injection, most often
in conjunction with either a supervised or home-based capsular stretching program.
Physical therapy that prioritizes toward rotator cuff strengthening is more appropriate
for patients with isolated subacromial impingement syndrome and may worsen
symptoms in patients with stiff shoulders. An MRI scan likely would not alter initial
treatment for patients who are stiff at presentation. Arthroscopic glenohumeral
capsular release is reserved for those patients who fail initial attempts at nonsurgical
management and remain functionally limited.
RECOMMENDED READINGS
Shoulder and Elbow Self-Assessment Examination AAOS 2014

1- Marx RG, Malizia RW, Kenter K, Wickiewicz TL, Hannafin JA. Intra-articular corticosteroid injection for the
treatment of idiopathic adhesive capsulitis of the shoulder. HSS J. 2007 Sep;3(2):202-7. doi: 10.1007/s11420-
007-9044-5. PubMed PMID: 18751795.View Abstract at PubMed
2- Johnson TS, Mesfin A, Farmer KW, McGuigan LA, Alamo IG, Jones LC, Johnson DC. Accuracy of intra-
articular glenohumeral injections: the anterosuperior technique with arthroscopic documentation. Arthroscopy.
2011 Jun;27(6):745-9. doi: 10.1016/j.arthro.2011.02.010. PubMed PMID: 21624668.View Abstract at PubMed
3- Shin SJ, Lee SY. Efficacies of corticosteroid injection at different sites of the shoulder for the treatment of
adhesive capsulitis. J Shoulder Elbow Surg. 2012 Sep 20. doi:pii: S1058-2746(12)00278-9.
10.1016/j.jse.2012.06.015. [Epub ahead of print] PubMed PMID: 22999847.View Abstract at PubMed
4- Neviaser AS, Hannafin JA. Adhesive capsulitis: a review of current treatment. Am J Sports Med. 2010
Nov;38(11):2346-56. doi: 10.1177/0363546509348048. Epub 2010 Jan 28. Review. PubMed PMID: 20110457.
View Abstract at PubMed

RESPONSES FOR QUESTIONS 79 THROUGH 82


1- Rotator cuff and scapular stabilizer strengthening exercises
2- Diagnostic and therapeutic corticosteroid injection
3- Arthroscopic debridement
4- Completion of rotator cuff tear, repair, and biceps tenotomy
5- Acromioplasty
6- Repair of rotator cuff and superior labrum anterior to posterior (SLAP) repair
7- Repair of subscapularis tendon and biceps tenodesis

Question 79 of 105
What is contraindicated in a patient with a partial articular supraspinatus tendon
avulsion lesion and the axial MRI scan shown in Figure 79?
1- Rotator cuff and scapular stabilizer strengthening exercises
2- Diagnostic and therapeutic corticosteroid
injection
3- Arthroscopic debridement
4- Completion of rotator cuff tear, repair, and
biceps tenotomy
5- Acromioplasty
6- Repair of rotator cuff and superior labrum
anterior to posterior (SLAP) repair
7- Repair of subscapularis tendon and biceps
tenodesis

PREFERRED RESPONSE: 5- Acromioplasty

Question 80 of 105
Shoulder and Elbow Self-Assessment Examination AAOS 2014

What is the most appropriate initial treatment in a 25-year-old professional baseball


player with a partial-thickness rotator cuff tear involving 40% thickness of the tendon?
1- Rotator cuff and scapular stabilizer strengthening exercises
2- Diagnostic and therapeutic corticosteroid injection
3- Arthroscopic debridement
4- Completion of rotator cuff tear, repair, and biceps tenotomy
5- Acromioplasty
6- Repair of rotator cuff and superior labrum anterior to posterior (SLAP) repair
7- Repair of subscapularis tendon and biceps tenodesis

PREFERRED RESPONSE: 1- Rotator cuff and scapular stabilizer strengthening


exercises

Question 81 of 105
What is the most appropriate definitive treatment in a 65-year-old man who has
experienced symptoms for more than 1 year and has a partial-thickness rotator cuff
tear involving 90% of the tendon and arthroscopy shown in Figure 81?
1- Rotator cuff and scapular stabilizer strengthening exercises
2- Diagnostic and therapeutic
corticosteroid injection
3- Arthroscopic debridement
4- Completion of rotator cuff tear, repair,
and biceps tenotomy
5- Acromioplasty
6- Repair of rotator cuff and superior
labrum anterior to posterior (SLAP)
repair
7- Repair of subscapularis tendon and
biceps tenodesis

PREFERRED RESPONSE: 4- Completion of rotator cuff tear, repair, and biceps


tenotomy

Question 82 of 105
What is the most appropriate treatment for a 25-year-old man 1 week after falling off a
ladder? His axial T2-weighted MRI scan is shown in Figure 82.
Shoulder and Elbow Self-Assessment Examination AAOS 2014

1- Rotator cuff and scapular stabilizer strengthening exercises


2- Diagnostic and therapeutic corticosteroid
injection
3- Arthroscopic debridement
4- Completion of rotator cuff tear, repair,
and biceps tenotomy
5- Acromioplasty
6- Repair of rotator cuff and superior labrum
anterior to posterior (SLAP) repair
7- Repair of subscapularis tendon and biceps
tenodesis

PREFERRED RESPONSE: 7- Repair of subscapularis tendon and biceps tenodesis


DISCUSSION
Acromioplasty can destabilize an os acromiale and is contraindicated. Initial treatment
of partial-thickness rotator cuff tears should be nonsurgical, with a focus on
rehabilitative exercises. Stiffness is more common after rotator cuff repair with
concomitant SLAP repair, and SLAP repair is not advocated in most people older than
40 years of age. Rotator cuff repair with biceps tenotomy or tenodesis is preferred to a
SLAP repair in this patient. Figure 82 shows a complete tear of the subscapularis
tendon with medial subluxation of the biceps tendon. In young patients, acute repair is
preferred with stabilization of the biceps tendon.
RECOMMENDED READINGS
1- Oh JH, Kim SH, Kwak SH, Oh CH, Gong HS. Results of concomitant rotator cuff and SLAP repair are not
affected by unhealed SLAP lesion. J Shoulder Elbow Surg. 2011 Jan;20(1):138-45. doi:
10.1016/j.jse.2010.04.003. Epub 2010 Jul 15. PubMed PMID: 20634099.View Abstract at PubMed
2- Wolff AB, Sethi P, Sutton KM, Covey AS, Magit DP, Medvecky M. Partial-thickness rotator cuff tears. J Am
Acad Orthop Surg. 2006 Dec;14(13):715-25. PubMed PMID: 17148619.View Abstract at PubMed

Question 83 of 105
Figures 83a and 83b are the radiographs of a 53-year-old otherwise healthy
homemaker who had a syncopal episode and sustained a ground-level fall and injury
to her right elbow. She presently admits to right elbow pain, swelling, and an inability
to bend her elbow. What is the best initial treatment for this injury?
Shoulder and Elbow Self-Assessment Examination AAOS 2014

1- Closed reduction with immobilization


2- Closed reduction with percutaneous pinning
3- Open reduction, bicolumnar fixation with plate and screws
4- Open reduction, bicolumnar fixation with Kirschner wires

PREFERRED RESPONSE: 3- Open reduction, bicolumnar fixation with plate and


screws
DISCUSSION
The radiographs and CT scans indicate a comminuted and displaced intra-articular
fracture of the distal humerus. Rigid internal fixation with bicolumnar orthogonal or
parallel plating is the treatment of choice for most fractures of the distal humerus that
involve the joint surface. Closed reduction and variations thereof will not yield a
stable environment for healing. To achieve adequate exposure for fixation, a chevron
olecranon osteotomy is the preferred approach. Disadvantages associated with this
approach include complications such as nonunion of the osteotomy site and intra-
articular adhesions. Prominent hardware may need to be removed during a secondary
procedure, and intraoperative conversion to an elbow arthroplasty may be limited. The
most common complications after open reduction and internal fixation include elbow
stiffness, nonunion (2%-10%), and ulnar neuropathy (0%-12%).
RECOMMENDED READINGS
1- Galano GJ, Ahmad CS, Levine WN. Current treatment strategies for bicolumnar distal humerus fractures. J Am
Acad Orthop Surg. 2010 Jan;18(1):20-30. Review. PubMed PMID: 20044489.View Abstract at PubMed
2- Ring D, Gulotta L, Chin K, Jupiter JB. Olecranon osteotomy for exposure of fractures and nonunions of the
distal humerus. J Orthop Trauma. 2004 Aug;18(7):446-9. PubMed PMID: 15289692.View Abstract at PubMed
3- Sanchez-Sotelo, J. Open reduction and internal fixation of supracondylar and intercondylar fractures. In: Wiesel
SW, ed. Operative Techniques in Orthopaedic Surgery. Philadelphia, PA: Lippincott Williams & Wilkins;
2011:3329-3336.

CLINICAL SITUATION FOR QUESTIONS 84 THROUGH 87


Figure 84 is the glenoid CT scan of a 20-year-old man who dislocated his shoulder
anteriorly while playing football. He had persistent instability 2 months after the
injury, but he did not have a sulcus sign or posterior instability. He underwent an
arthroscopic Bankart repair with 4 bioabsorbable anchors with simple sutures through
the labrum and capsule. He did not have an engaging Hill-Sachs lesion, the rotator
cuff was unremarkable, and the capsule was not torn from the humerus. After surgery,
he did well for 6 months until he jumped into a lake and again dislocated his shoulder
anteriorly. He says his shoulder no longer felt stable after his reduction.
Shoulder and Elbow Self-Assessment Examination AAOS 2014

Question 84 of 105
What is the most likely reason this patient’s
arthroscopic Bankart repair failed?
1- The surgeon did not use enough
anchors to repair the labrum.
2- The surgeon did not recognize
significant bone loss of the anterior
glenoid.
3- The patient returned to full activity too
soon.
4- The patient has unrecognized multidirectional instability.

PREFERRED RESPONSE: 2- The surgeon did not recognize significant bone loss
of the anterior glenoid.

Question 85 of 105
This patient would like to return to football and perform normal activities of daily
living without worrying about another dislocation. What treatment would you
recommend?
1- Open Bankart repair
2- Coracoid transfer
3- Revision arthroscopic labrum repair
4- Arthroscopic pan capsular plication and labrum repair

PREFERRED RESPONSE: 2- Coracoid transfer

Question 86 of 105
What is the most common early complication of the revision procedure for this
patient?
1- Loss of external rotation
2- Loss of internal rotation
3- Recurrent instability
4- Subscapularis tear

PREFERRED RESPONSE: 1- Loss of external rotation


Shoulder and Elbow Self-Assessment Examination AAOS 2014

Question 87 of 105
What is the most common late complication of the revision procedure for this patient?
1- Glenohumeral arthritis
2- Bone graft absorption
3- Anterior ligament attenuation
4- Rotator cuff tear

PREFERRED RESPONSE: 1- Glenohumeral arthritis


DISCUSSION
The CT scan shows bone loss exceeding 20% on the anterior glenoid, which is the
most likely reason the arthroscopic Bankart repair failed. One study showed that using
3 or fewer anchors increases risk for failure; 4 anchors were used in this patient, so
that is not the likely cause of failure. The patient returned to full activity 6 months
after surgery, which is the usual time needed to regain full strength in the shoulder and
ensure complete labrum healing. Suture configuration has not been shown to affect
failure rates. The patient did not have signs of multidirectional instability such as a
sulcus sign on examination, instability without a labral tear, or excessive translation of
the humeral head posteriorly on examination. This patient has recurrent instability due
to glenoid bone loss, so the procedure of choice would need to restore the anterior
bone to the glenoid. The coracoid transfer procedure uses the coracoid for bone
restoration, but iliac crest bone graft would be appropriate as well. An open Bankart
repair, arthroscopic capsular plication, or a revision arthroscopic repair are all soft-
tissue procedures, which do not correct the bone loss. Braces may work to allow a
patient to finish a season before having surgery, but will not allow a return to activities
of daily living without instability. The most common complication of the coracoid
transfer is a loss of external rotation. The rate of recurrent instability is low. Most
patients regain all of their internal rotation. The technique for the coracoid transfer
splits the subscapularis muscle, so a tear of the muscle is rare. Axillary or
musculocutaneous nerve palsies are rare after this procedure, but can occur if the
nerves are not protected and mobilized during the dissection of the conjoint tendon.
The most common long-term complication is early arthritis of the glenohumeral joint.
Most cases of arthritis are asymptomatic and appear on follow-up radiographs. The
graft rarely absorbs, and tears of the rotator cuff are uncommon with this procedure
and infection is rare. Anterior ligament attenuation is uncommon, and some surgeons
do not even repair the anterior labrum or capsule because this can lead to a loss of
external rotation after surgery.
RECOMMENDED READINGS
Shoulder and Elbow Self-Assessment Examination AAOS 2014

1- Burkhart SS, De Beer JF. Traumatic glenohumeral bone defects and their relationship to failure of arthroscopic
Bankart repairs: significance of the inverted-pear glenoid and the humeral engaging Hill-Sachs lesion.
Arthroscopy. 2000 Oct;16(7):677-94. PubMed PMID: 11027751.View Abstract at PubMed
2- Ochoa E Jr, Burkhart SS. Glenohumeral bone defects in the treatment of anterior shoulder instability. Instr
Course Lect. 2009;58:323-36. PubMed PMID: 19385546.View Abstract at PubMed
3- Schmid SL, Farshad M, Catanzaro S, Gerber C. The Latarjet procedure for the treatment of recurrence of
anterior instability of the shoulder after operative repair: a retrospective case series of forty-nine consecutive
patients. J Bone Joint Surg Am. 2012 Jun 6;94(11):e75. doi: 10.2106/JBJS.K.00380. PubMed PMID: 22637215.
View Abstract at PubMed
4- Neyton L, Young A, Dawidziak B, Visona E, Hager JP, Fournier Y, Walch G. Surgical treatment of anterior
instability in rugby union players: clinical and radiographic results of the Latarjet-Patte procedure with
minimum 5-year follow-up. J Shoulder Elbow Surg. 2012 Dec;21(12):1721-7. doi: 10.1016/j.jse.2012.01.023.
Epub 2012 May 5. PubMed PMID: 22565042.View Abstract at PubMed
5- Hovelius L, Vikerfors O, Olofsson A, Svensson O, Rahme H. Bristow-Latarjet and Bankart: a comparative
study of shoulder stabilization in 185 shoulders during a seventeen-year follow-up. J Shoulder Elbow Surg.
2011 Oct;20(7):1095-101. doi: 10.1016/j.jse.2011.02.005. Epub 2011 May 24. PubMed PMID: 21602067.
View Abstract at PubMed
6- Hovelius L, Saeboe M. Neer Award 2008: Arthropathy after primary anterior shoulder dislocation--223
shoulders prospectively followed up for twenty-five years. J Shoulder Elbow Surg. 2009 May-Jun;18(3):339-47.
doi: 10.1016/j.jse.2008.11.004. Epub 2009 Feb 28. PubMed PMID: 19254851.View Abstract at PubMed

Question 88 of 105
Complete transection of the ulnar nerve at the elbow will result in
1- loss of sensation on the ulnar side of the index finger.
2- weakness with thumb extension.
3- weakness with elbow flexion.
4- weakness with finger abduction.

PREFERRED RESPONSE: 4- weakness with finger abduction.


DISCUSSION
Ulnar nerve lesions manifest with weakness in the finger abductor muscles. There will
be loss of interossei muscle function as well as the third and fourth lumbricals.
Extensor pollicis longus function is based on the posterior interosseous nerve (radial),
not the ulnar. The index finger has sensation from the median nerve distribution.
Elbow flexion strength is not dependent on the ulnar nerve.
RECOMMENDED READINGS
1- Hoppenfeld S. Physical Examination of the Spine and Extremities. New York: Appleton-Century-Crofts;
1976:93-104.
2- Netter FH. The CIBA Collection of Medical Illustrations. Musculoskeletal System Part 1. Vol 8. Summit, NJ:
Ciba-Geigy Corporation; 1987:28.
Shoulder and Elbow Self-Assessment Examination AAOS 2014

Question 89 of 105
Figures 89a and 89b are the radiograph and MRI scan of a 40-year-old man who fell
down a flight of stairs. His upper arm is bruised and painful, and global weakness in
the shoulder girdle function is noted. A radiograph is ordered to rule out a fracture or
dislocation. You should recommend

1- immediate open reduction and internal fixation of the fracture.


2- closed treatment with serial radiographs.
3- fracture fragment excision and deltoid repair.
4- rest, ice, anti-inflammatory medications, and a home exercise program.

PREFERRED RESPONSE: 4- rest, ice, anti-inflammatory medications, and a home


exercise program.
DISCUSSION
The patient has an os acromiale. The type shown is of the meso-acromion. This is not
an acute fracture; well corticated ends are seen on the axillary radiograph and there is
no bone edema on the T2 axial MRI image. A trial of nonsurgical care that includes
rest, ice, and anti-inflammatory medication is recommended. If a patient continues to
have symptoms, an arthroscopic evaluation is needed to determine if the os is mobile
and if os fixation is appropriate.
RECOMMENDED READINGS
1- Sammarco VJ. Os acromiale: frequency, anatomy, and clinical implications. J Bone Joint Surg Am. 2000
Mar;82(3):394-400.View Abstract at PubMed
2- Harris JD, Griesser MJ, Jones GL. Systematic review of the surgical treatment for symptomatic os acromiale.
Int J Shoulder Surg. 2011 Jan;5(1):9-16.View Abstract at PubMed
3- Abboud JA, Silverberg D, Pepe M, Beredjiklian PK, Iannotti JP, Williams GR, Ramsey ML. Surgical treatment
of os acromiale with and without associated rotator cuff tears. J Shoulder Elbow Surg. 2006 May-Jun;15(3):265-
70.View Abstract at PubMed
Shoulder and Elbow Self-Assessment Examination AAOS 2014

Question 90 of 105
Figure 90 is the initial radiograph of a 28-year-old woman who sustained an acute
right elbow injury. Following closed treatment under sedation in the emergency
department, the elbow is seen to be stable through an arc
from full flexion down to 30 degrees short of full
extension, while the forearm is pronated but only to 75
degrees short of full extension while in supination. What
structure is most likely to remain intact?

1- Lateral ulnar collateral ligament


2- Radial head
3- Posterior band of the medial collateral ligament
(MCL)
4- Anterior band of the MCL

PREFERRED RESPONSE: 4- Anterior band of the MCL


DISCUSSION
The most common pattern of elbow dislocation is associated with posterolateral
rotatory instability. This pattern begins with valgus, axial load, and supination rotating
the radial head posterior with respect to the capitellum and failure of the lateral ulnar
collateral ligament. The posterior band of the MCL tears next, and the anterior band of
the MCL is last to fail. In elbows with an intact anterior band of the MCL, forearm
pronation will place this structure under tension and assist in maintaining joint
reduction. If this band is torn, pronation will lead to medial joint space widening.
Radial head fractures, along with coronoid fractures, are common associated injuries,
as seen in the radiographs for this patient.
RECOMMENDED READINGS
1- O'Driscoll SW, Morrey BF, Korinek S, An KN. Elbow subluxation and dislocation. A spectrum of instability.
Clin Orthop Relat Res. 1992 Jul;(280):186-97.View Abstract at PubMed
2- Olsen BS, Søjbjerg JO, Nielsen KK, Vaesel MT, Dalstra M, Sneppen O. Posterolateral elbow joint instability:
the basic kinematics. J Shoulder Elbow Surg. 1998 Jan-Feb;7(1):19-29View Abstract at PubMed
3- O'Driscoll SW. Classification and evaluation of recurrent instability of the elbow. Clin Orthop Relat Res. 2000
Jan;(370):34-43. Review.View Abstract at PubMed

Question 91 of 105
Figures 91a through 91d are the radiographs of an 86-year-old man who lives
independently who has fallen down the stairs. He has an isolated elbow injury. What
treatment option is most likely to offer the most rapid return of function and pain
relief?
Shoulder and Elbow Self-Assessment Examination AAOS 2014

1- Open reduction and internal fixation


(ORIF)
2- Cast treatment for 4 weeks followed
by static splinting
3- Percutaneous pinning
4- Total elbow arthroplasty (TEA)

PREFERRED RESPONSE: 4- Total elbow arthroplasty (TEA)


DISCUSSION
Advantages of TEA for fracture in elderly patients include preservation of the extensor
mechanism, early mobilization, and avoiding complications of fixation; however,
there are lifetime activity limitations and risk for loosening over time. Reoperation
rates may be lower, functional outcomes improved, and results more predictable for
TEA than ORIF in elderly populations. Cast treatment leads to unacceptable rates of
stiffness and disability. Percutaneous fixation does not provide rigid enough fixation
in the adult population.
RECOMMENDED READINGS
1- Sanchez-Sotelo J. Fractures of the distal humerus. In: Galatz L, ed. Orthopaedic Knowledge Update Shoulder
and Elbow 3. Rosemont, IL: American Academy of Orthopaedic Surgeons; 2008:507-516.
2- Sanchez-Sotelo J. Distal humeral fractures: role of internal fixation and elbow arthroplasty. Instr Course Lect.
2012;61:203-13.View Abstract at PubMed
3- McKee MD, Veillette CJ, Hall JA, Schemitsch EH, Wild LM, McCormack R, Perey B, Goetz T, Zomar M,
Moon K, Mandel S, Petit S, Guy P, Leung I. A multicenter, prospective, randomized, controlled trial of open
reduction--internal fixation versus total elbow arthroplasty for displaced intra-articular distal humeral fractures
Shoulder and Elbow Self-Assessment Examination AAOS 2014

in elderly patients. J Shoulder Elbow Surg. 2009 Jan-Feb;18(1):3-12. doi: 10.1016/j.jse.2008.06.005. Epub 2008
Sep 26.View Abstract at PubMed
4- Kim JM, Mudgal CS, Konopka JF, Jupiter JB. Complications of total elbow arthroplasty. J Am Acad Orthop
Surg. 2011 Jun;19(6):328-39. Review.View Abstract at PubMed

Question 92 of 105
A 68-year-old right-hand-dominant man underwent a right total shoulder arthroplasty
(TSA) 3 months ago. He was started on passive range of motion and started active
motion 6 weeks after surgery. He notes that he fell onto his outstretched right arm 2
weeks ago but did not seek care. His primary symptom is poor active elevation of the
right shoulder. His right shoulder motion has active elevation of 45 degrees, passive
elevation of 140 degrees, 95-degree external rotation, and internal rotation to L3. His
left shoulder has active and passive elevation of 160 degrees, external rotation of 70
degrees, and internal rotation to T12. The right shoulder radiographs show a
concentric total shoulder arthroplasty with no fractures or other abnormalities. What is
the most appropriate treatment at this point?
1- Reassurance and a review of his rehabilitation program with an emphasis on
deltoid strengthening
2- Open repair of the subscapularis tendon
3- Latissimus dorsi tendon transfer
4- Revision to reverse TSA

PREFERRED RESPONSE: 2- Open repair of the subscapularis tendon


DISCUSSION
This patient had a fall approximately 2½ months following a TSA. He now has poor
active elevation but good passive motion and external rotation exceeding that of the
contralateral shoulder. Rupture of the subscapularis tendon, which would have been
released and repaired intrasurgically, would be the primary concern in this scenario. In
the native shoulder treated surgically for instability, subscapularis failure can produce
pain, weakness of abdominal press and lumbar pushoff, apprehension, and frank
instability. Further delay in treatment of the tendon failure with therapy is not
indicated because this will lead to further muscle atrophy and adhesions to the scapula
and overlying brachial plexus. Augmentation or replacement with a transfer of the
superior portion of the pectoralis major muscle is sometimes required. Transfer of the
pectoralis minor muscle is also described. However, latissimus dorsi transfer is
described for irreparable supraspinatus deficiency. Revision to a reverse TSA can be
considered as a salvage of a persistently unstable shoulder, but will not be the primary
treatment for this shoulder if radiograph findings are normal.
Shoulder and Elbow Self-Assessment Examination AAOS 2014

RECOMMENDED READINGS
1- Lazarus MD, Harryman DT II. Open repairs for anterior instability. In: Warner J, Iannotti J, Gerver R, eds.
Complex and Revision Problems in Shoulder Surgery. Philadelphia, PA: Lippincott-Raven; 1996:47-63.
2- Sanchez-Sotelo J, Sperling JW, Rowland CM, Cofield RH. Instability after shoulder arthroplasty: results of
surgical treatment. J Bone Joint Surg Am. 2003 Apr;85-A(4):622-31.View Abstract at PubMed
3- Moeckel BH, Altchek DW, Warren RF, Wickiewicz TL, Dines DM. Instability of the shoulder after
arthroplasty. J Bone Joint Surg Am. 1993 Apr;75(4):492-7.View Abstract at PubMed
4- Miller BS, Joseph TA, Noonan TJ, Horan MP, Hawkins RJ. Rupture of the subscapularis tendon after shoulder
arthroplasty: diagnosis, treatment, and outcome. J Shoulder Elbow Surg. 2005 Sep-Oct;14(5):492-6.View
Abstract at PubMed

Question 93 of 105
Figure 93 is the radiograph of a 72-year-old woman.
Treatment includes fixation of the ulna. What
options are recommended for the radius?

1- Radial head replacement to restore


radiocapitellar contact
2- Radial head excision because there is no
risk for posterolateral instability
3- Percutaneous fixation to avoid the risk for
stiffness after surgery
4- Allograft reconstruction to prevent
capitellar erosion

PREFERRED RESPONSE: 1- Radial head replacement to restore radiocapitellar


contact
DISCUSSION
Prosthetic replacement is an appropriate option in cases of a nonreconstructable
fracture to restore the radiocapitellar contact. Most complex fractures are associated
with instability; therefore, it is advisable to consider open reduction and internal
fixation or radial head replacement when the injury involves a dislocation or fracture
of the ulna. Simple radial head excision may be a viable option for a comminuted
fracture without instability or associated ulnar fracture. When the radial head is
replaced, caution must be exercised to avoid overstuffing the joint because this can
lead to stiffness from impingement, capitellar erosion, loss of flexion, or synovitis.
RECOMMENDED READINGS
1- Ring D. Radial head fractures. In: Galatz L, ed. Orthopaedic Knowledge Update Shoulder and Elbow 3.
Rosemont, IL: American Academy of Orthopaedic Surgeons; 2008:497-506.
2- Rosenblatt Y, Athwal GS, Faber KJ. Current recommendations for the treatment of radial head fractures. Orthop
Clin North Am. 2008 Apr;39(2):173-85, vi. doi: 10.1016/j.ocl.2007.12.008. Review.View Abstract at PubMed
Shoulder and Elbow Self-Assessment Examination AAOS 2014

3- Duckworth AD, Watson BS, Will EM, Petrisor BA, Walmsley PJ, Court-Brown CM, McQueen MM. Radial
head and neck fractures: functional results and predictors of outcome. J Trauma. 2011 Sep;71(3):643-8. doi:
10.1097/TA.0b013e3181f8fa5f.View Abstract at PubMed
4- Herbertsson P, Josefsson PO, Hasserius R, Karlsson C, Besjakov J, Karlsson M; Long-Term Follow-Up Study.
Uncomplicated Mason type-II and III fractures of the radial head and neck in adults. A long-term follow-up
study. J Bone Joint Surg Am. 2004 Mar;86-A(3):569-74.View Abstract at PubMed

CLINICAL SITUATION FOR QUESTIONS 94 THROUGH 96


Figure 94 is the anteroposterior radiograph of a 75-year-old woman who has a 5-year
history of progressive pain, crepitus, and loss of motion in her shoulder. She had a
rotator cuff repair 10 years ago. Examination reveals 60 degrees of active forward
elevation and 20 degrees of external rotation with her arm at her side. Passively she
can be brought to 160 degrees of forward elevation and 90 degrees of external rotation
with her arm at her side. A glenohumeral joint injection with local anesthetic
eliminated pain, but there is no observed change in active motion.

Question 94 of 105
Based upon the information provided, you should
recommend
1- total shoulder arthroplasty (TSA).
2- arthroscopic rotator cuff repair.
3- arthroscopic debridement.
4- reverse total shoulder arthroplasty (rTSA).

PREFERRED RESPONSE: 4- reverse total shoulder arthroplasty (rTSA).

Question 95 of 105
Your treatment decision is the best option because the
1- prosthesis is designed to convert the translational force of the deltoid to
rotational motion.
2- use of an all-polyethylene glenoid component will reduce risk for developing
glenoid pain after humeral head arthroplasty.
3- poor motion is a function of synovitis.
4- weakness is generated from pain.

PREFERRED RESPONSE: 1- prosthesis is designed to convert the translational


force of the deltoid to rotational motion.
Shoulder and Elbow Self-Assessment Examination AAOS 2014

Question 96 of 105
A common postoperative radiographic observation associated with your surgery in an
asymptomatic patient is
1- implant fracture.
2- suture anchor dislodgement.
3- scapular notching.
4- acromial fracture.

PREFERRED RESPONSE: 3- scapular notching.


DISCUSSION
The patient has anterior superior instability. This condition is caused by a combination
of rotator cuff insufficiency and loss of coracoacromial arch integrity. The only known
way to restore shoulder function in this scenario is to implant a rTSA. The device
works by converting the translational force of the deltoid into a rotational force,
resulting in restoration of forward elevation of the shoulder. Performing a rotator cuff
repair or arthroscopic debridement will not address this biomechanical problem. TSA
will also not change this biomechanical problem. The poor motion and function are
not a result of synovitis or pain because an injection with local anesthetic has
eliminated the pain and serves as a useful test to determine if rTSA is the only viable
solution. If the patient can achieve near-normal function with a local anesthetic
challenge, rTSA is overtreatment. Scapular notching is a long-term concern for
implant longevity because it represents bone loss under the baseplate of the glenoid
component. This loss of support can lead to catastrophic failure of the device. Implant
fracture, acromial fracture, and dislodgement of suture anchors are not likely to be
asymptomatic in a non-Charcot joint.

RECOMMENDED READINGS
1- Melis B, DeFranco M, Ladermann A, Mole D, Favard L, Nerot C, Maynou C, Walch G. An evaluation of the
radiological changes around the Grammont reverse geometry shoulder arthroplasty after eight to 12 years. J
Bone Joint Surg Br. 2011 Sep;93(9): 1240-6. PubMed PMID: 21911536View Abstract at PubMed
2- Leung B, Horodyski M, Struk AM, Wright TW. Functional outcome of hemiarthroplasty compared with reverse
total shoulder arthroplasty in the treatment of rotator cuff tear arthropathy. J Shoulder Elbow Surg. 2012
Mar;21(3):319-23. Epub 2011 Aug 26.View Abstract at PubMed
3- Cheung E, Willis M, Walker M, Clark R, Frankle MA. Complications in reverse total shoulder arthroplasty. J
Am Acad Orthop Surg. 2011 Jul;19(7):439-49. Review.View Abstract at PubMed
4- Walker M, Brooks J, Willis M, Frankle M. How reverse shoulder arthroplasty works. Clin Orthop Relat Res.
2011 Sep;469(9):2440-51. Review.View Abstract at PubMed
5- Sadoghi P, Vavken P, Leithner A, Hochreiter J, Weber G, Pietschmann MF, Müller PE. Impact of previous
rotator cuff repair on the outcome of reverse shoulder arthroplasty. J Shoulder Elbow Surg. 2011
Oct;20(7):1138-46. Epub 2011 Mar 30.View Abstract at PubMed
Shoulder and Elbow Self-Assessment Examination AAOS 2014

6- Nam D, Kepler CK, Neviaser AS, Jones KJ, Wright TM, Craig EV, Warren RF. Reverse total shoulder
arthroplasty: current concepts, results, and component wear analysis. J Bone Joint Surg Am. 2010 Dec;92 Suppl
2:23-35.View Abstract at PubMed

Question 97 of 105
A 36-year-old woman dislocated her elbow 6 months ago. The elbow was congruently
reduced and rehabilitated. She continues to have a sense of painful clunking in her
elbow when she pushes up from a chair with forearm supination, but not pronation.
What structure did not heal properly?
1- Posterior band of the medial collateral ligament
2- Anterior band of the medial collateral ligament
3- Radial collateral ligament
4- Lateral ulnar collateral ligament

PREFERRED RESPONSE: 4- Lateral ulnar collateral ligament

DISCUSSION
The patient is showing signs of posterolateral rotatory instability after elbow
dislocation. The lateral ulnar collateral ligament is responsible for stabilizing the
elbow against this type of instability. The posterior and anterior bands of the medial
collateral ligament are primarily resistors of valgus load in elbow extension and
flexion, respectively. The radial collateral ligament does not control the posterolateral
rotatory instability described.
RECOMMENDED READINGS
1- O'Driscoll SW, Morrey BF, Korinek S, An KN. Elbow subluxation and dislocation. A spectrum of instability.
Clin Orthop Relat Res. 1992 Jul;(280):186-97View Abstract at PubMed
2- O'Driscoll SW, Bell DF, Morrey BF. Posterolateral rotatory instability of the elbow. J Bone Joint Surg Am.
1991 Mar;73(3):440-6.View Abstract at PubMed

Question 98 of 105
What complication following total elbow arthroplasty poses more risk for a 60-year-
old man with osteoarthritis than for a man of the same age with rheumatoid arthritis?
1- Aseptic loosening of a linked implant
2- Instability of an unlinked implant
3- Triceps rupture
4- Wound dehiscence

PREFERRED RESPONSE: 1- Aseptic loosening of a linked implant


Shoulder and Elbow Self-Assessment Examination AAOS 2014

DISCUSSION
Patients with elbow osteoarthritis tend to be active and are often involved in manual
occupations that place greater demands on a total elbow implant. Such patients are
most often treated with nonprosthetic options because of concerns about prosthetic
longevity. As a result, few cases of primary osteoarthritis are included in published
studies. However, complications such as stem fracture and aseptic loosening appear to
be more common in this population than in any other subgroup, including revision
patients. The poor soft-tissue quality associated with rheumatoid arthritis leads to a
high-risk ligamentous attenuation and is a general contraindication to use of an
unlinked implant. The same poor soft tissue leads to a higher rate of triceps
insufficiency and wound dehiscence.
RECOMMENDED READINGS
1- Kozak TKW, Adams RA, Morrey BF. Total elbow arthroplasty for primary osteoarthritis. In: Morrey BF,
Sanchez-Sotelo J, eds. The Elbow and Its Disorders. Philadelphia, PA: WB Saunders; 2009:843-848.
2- Gill DRJ, Morrey BF, Adams RA. Linked total elbow arthroplasty in patients with rheumatoid arthritis. Total
elbow arthroplasty for primary osteoarthritis. In: Morrey BF, Sanchez-Sotelo J, eds. The Elbow and Its
Disorders. Philadelphia, PA: WB Saunders; 2009:782-791.
3- Gramstad GD, Galatz LM. Management of elbow osteoarthritis. J Bone Joint Surg Am. 2006 Feb;88(2):421-30.
Review.View Abstract at PubMed
4- Gill DR, Morrey BF. The Coonrad-Morrey total elbow arthroplasty in patients who have rheumatoid arthritis. A
ten to fifteen-year follow-up study. J Bone Joint Surg Am. 1998 Sep;80(9):1327-35.View Abstract at PubMed

Question 99 of 105
Figure 99a is the radiograph of a 48-year-old woman 8 months after initial treatment
of an injury. She initially was placed in a sling and progressive rehabilitation followed.
She now has refractory pain but normal range of movement and strength. The current
radiograph is shown in Figure 99b. The most appropriate next treatment step is

1- Resumption of sling immobilization


2- Open reduction and internal fixation
3- Application of a bone stimulator and rest from exacerbating activities
4- Application of a figure-of-8 brace
Shoulder and Elbow Self-Assessment Examination AAOS 2014

PREFERRED RESPONSE: 2- Open reduction and internal fixation


DISCUSSION
The radiographs reveal an atrophic nonunion of the midshaft of the clavicle. The
treatment of acute, displaced midshaft clavicle fractures in adults continues to evolve,
with several reports advocating early surgical intervention. Although many fractures
heal, symptomatic delayed unions or nonunions occur and may eventually require
treatment. In this case, further sling immobilization or use of a figure-of-8 brace is
unlikely to lead to fracture consolidation at 8 months after the injury. Although use of
an electrical bone stimulator may be attractive, there is no conclusive data suggesting
its efficacy in promoting healing of a displaced clavicular nonunion. Most authors
advocate treatment with open reduction internal plate fixation. Controversy exists as to
the need for allograft or autograft bone augmentation.
RECOMMENDED READINGS
1- van der Meijden OA, Gaskill TR, Millett PJ. Treatment of clavicle fractures: current concepts review. J
Shoulder Elbow Surg. 2012 Mar;21(3):423-9. doi: 10.1016/j.jse.2011.08.053. Epub 2011 Nov 6. Review.
View Abstract at PubMed
2- Khan LA, Bradnock TJ, Scott C, Robinson CM. Fractures of the clavicle. J Bone Joint Surg Am. 2009
Feb;91(2):447-60. doi: 10.2106/JBJS.H.00034. Review.View Abstract at PubMed
3- Endrizzi DP, White RR, Babikian GM, Old AB. Nonunion of the clavicle treated with plate fixation: a review of
forty-seven consecutive cases. J Shoulder Elbow Surg. 2008 Nov-Dec;17(6):951-3. doi:
10.1016/j.jse.2008.05.046. Epub 2008 Sep 20. View Abstract at PubMed

Question 100 of 105


A 75-year-old woman sustained a 4-part fracture dislocation of the proximal humerus
with a comminuted humeral head. You decide to perform a reverse total shoulder
replacement because of her age and activity level. This will be your first reverse total
shoulder replacement. It is common practice in your hospital for an industry
representative to be present when new implants are brought into the operating room.
What information are you required to disclose?
1- This is an experimental procedure.
2- You have no financial relationship with the implant company.
3- There will be an implant company representative in the room.
4- The hospital will lose money because of the cost of the implant.

PREFERRED RESPONSE: 3- There will be an implant company representative in


the room.
DISCUSSION
Current recommendations from the American Orthopaedic Association Orthopaedic
Institute of Medicine are that the patient be notified if an industry representative is
Shoulder and Elbow Self-Assessment Examination AAOS 2014

going to be present in the operating room. This surgery is not experimental for this
indication, and Medicare currently covers the surgery for patients with appropriate
indications. Court cases have demonstrated that surgeon-related factors can be
litigated (such as surgeon experience), but there are no current requirements to
disclose this. Surgeons are not required to disclose cost and compensation information
to their patients.
RECOMMENDED READINGS
1- Report from the task force on surgeon-industry relationships in the discipline of orthopaedic surgery. American
Orthopaedic Association Orthopaedic Institute of Medicine Task Force on Surgeon-Industry Relationships in
the Discipline of Orthopaedic Surgery. J Bone Joint Surg Am. 2012 Jun 20;94(12):e89.View Abstract at
PubMed
2- Bal BS, Choma TJ. Clin Orthop Relat Res. What to disclose? Revisiting informed consent. 2012
May;470(5):1346-56.View Abstract at PubMed

RESPONSES FOR QUESTIONS 101 THROUGH 104


1- Arthroscopic or open debridement and capsular release
2- Interposition arthroplasty
3- Ulnohumeral arthrodesis
4- Linked total elbow arthroplasty (TEA)
5- Unlinked TEA

What surgical procedure listed above is most associated with the conditions
defined below?

Question 101 of 105


This procedure has implant or graft loosening as the main postsurgical concern.
1- Arthroscopic or open debridement and capsular release
2- Interposition arthroplasty
3- Ulnohumeral arthrodesis
4- Linked total elbow arthroplasty (TEA)
5- Unlinked TEA

PREFERRED RESPONSE: 4- Linked total elbow arthroplasty (TEA)

Question 102 of 105


This procedure is associated with progressive resorption of distal humeral condyles
after surgery.
1- Arthroscopic or open debridement and capsular release
2- Interposition arthroplasty
Shoulder and Elbow Self-Assessment Examination AAOS 2014

3- Ulnohumeral arthrodesis
4- Linked total elbow arthroplasty (TEA)
5- Unlinked TEA

PREFERRED RESPONSE: 2- Interposition arthroplasty

Question 103 of 105


A 50-year-old woman with poorly controlled rheumatoid arthritis has advanced
destruction of the right-dominant elbow.
1- Arthroscopic or open debridement and capsular release
2- Interposition arthroplasty
3- Ulnohumeral arthrodesis
4- Linked total elbow arthroplasty (TEA)
5- Unlinked TEA

PREFERRED RESPONSE: 4- Linked total elbow arthroplasty (TEA)

Question 104 of 105


A 41-year-old man who is a carpenter has moderate posttraumatic arthritis and pain in
terminal extension affecting function.
1- Arthroscopic or open debridement and capsular release
2- Interposition arthroplasty
3- Ulnohumeral arthrodesis
4- Linked total elbow arthroplasty (TEA)
5- Unlinked TEA

PREFERRED RESPONSE: 1- Arthroscopic or open debridement and capsular


release

DISCUSSION
Linked prosthetic TEA has been a common choice for surgical treatment of elbow
arthritis, and reports document good results in many patients. However, load on the
bearing surfaces and on the implant/cement/ bone interface are sources of failure,
particularly in younger and higher-demand patients. The use of nonprosthetic options
such as arthroscopic debridement or interposition arthroplasty is advocated in this
population. Unlinked arthroplasties have been developed in an effort to reduce stem
loosening by decreasing the constraint of the articulation.
Shoulder and Elbow Self-Assessment Examination AAOS 2014

In contrast, patients with rheumatoid arthritis often have attenuated ligamentous


support and may develop instability with the same unlinked implants. Patients with
rheumatoid arthritis also typically have lower demands because of polyarticular
disease. Linked TEA has been shown to have similar survival as total hip replacement
in this population.
Bone resorption is an occasional complication of interposition arthroplasty. This can
lead to subluxation and poor outcome, particularly if it occurs more on one side than
the other.
Unlinked TEA would be considered in severe, concentric arthritis with intact
ligamentous support in young, active patients. Ulnohumeral arthrodesis has poor
functional outcome and rarely is considered a salvage procedure.
RECOMMENDED READINGS
1- Kozak TKW, Adams RA, Morrey BF. Total elbow arthroplasty for primary osteoarthritis. In: Morrey BF,
Sanchez-Sotelo J, eds. The Elbow and Its Disorders. Philadelphia, PA: WB Saunders; 2009:843-849.
2- Gill DRJ, Morrey BF, Adams RA: Linked total elbow arthroplasty in patients with rheumatoid arthritis. In:
Morrey BF, Sanchez-Sotelo J, eds. The Elbow and Its Disorders. Philadelphia, PA: WB Saunders; 2009:782-
791.
3- Gramstad GD, Galatz LM. Management of elbow osteoarthritis. J Bone Joint Surg Am. 2006 Feb;88(2):421-30.
Review.View Abstract at PubMed
4- Grishin IG, Goncharenko IV, Kozhin NP, Sarkisyan AG, Gobulev AG, Devis AE. Restoration of the function of
the cubital joint in extensive defects of bones and soft tissues using endoprosthesis and free skin grafts. Acta
Chir Plast. 1989;31(3):143-7.View Abstract at PubMed
5- Gill DR, Morrey BF. The Coonrad-Morrey total elbow arthroplasty in patients who have rheumatoid arthritis. A
ten to fifteen-year follow-up study. J Bone Joint Surg Am. 1998 Sep;80(9):1327-35.View Abstract at PubMed

Question 105 of 105


Figures 105a and 105b are the radiograph and MRI scan of a 45-year-old woman with
fibromyalgia that causes chronic neck and scapula pain. She has had new-onset lateral
shoulder pain for 1 year. She has tenderness throughout her shoulder, back, and neck;
a positive Hawkins impingement sign; and pain with resisted elevation. She tried
physical therapy for 12 weeks and the pain is worse. What is the next appropriate
treatment step for her shoulder pain?
Shoulder and Elbow Self-Assessment Examination AAOS 2014

1- Subacromial injection
2- Referral to pain management
3- Arthroscopic removal of a calcium deposit
4- Arthroscopic subacromial decompression

PREFERRED RESPONSE: 1- Subacromial injection


DISCUSSION
The radiographs show a calcium deposit in the supraspinatus tendon. The MRI scan
shows a small black signal in the supraspinatus tendon without a tear of the tendon.
These findings are consistent with calcific tendonitis. A subacromial injection will
help distinguish between the pain from the calcific tendonitis and the chronic
fibromyalgia. Pain management is an option after an injection is done for diagnostic
purposes. Surgery should not be considered until the diagnosis is confirmed because it
will not be successful if the shoulder pain is attributable to the patient’s fibromyalgia.
RECOMMENDED READINGS
1- Hurt G, Baker CL Jr. Calcific tendinitis of the shoulder. Orthop Clin North Am. 2003 Oct;34(4):567-75.
Review.View Abstract at PubMed
2- Porcellini G, Paladini P, Campi F, Paganelli M. Arthroscopic treatment of calcifying tendinitis of the shoulder:
clinical and ultrasonographic follow-up findings at two to five years. J Shoulder Elbow Surg. 2004 Sep-
Oct;13(5):503-8View Abstract at PubMed

You might also like